SlideShare a Scribd company logo
1 of 32
Download to read offline
53
C H A P T E R
3
Second-Order Linear Equations
3.1
1. Let y = ert
, so that y = r ert
and y = r2
ert
. Direct substitution into the
differential equation yields (r2
+ 2r − 3)ert
= 0 . Canceling the exponential, the
characteristic equation is r2
+ 2r − 3 = 0 . The roots of the equation are r = −3 , 1 .
Hence the general solution is y = c1et
+ c2e−3t
.
2. Let y = ert
. Substitution of the assumed solution results in the characteristic
equation r2
+ 3r + 2 = 0 . The roots of the equation are r = −2 , −1 . Hence the
general solution is y = c1e−t
+ c2e−2t
.
5. The characteristic equation is 4r2
− 9 = 0 , with roots r = ±3/2 . Therefore the
general solution is y = c1e−3t/2
+ c2e3t/2
.
6. The characteristic equation is r2
− 2r − 2 = 0 , with roots r = 1 ±
√
3 . Hence
the general solution is y = c1e(1−
√
3)t
+ c2e(1+
√
3)t
.
7. Substitution of the assumed solution y = ert
results in the characteristic equa-
tion r2
+ r − 2 = 0 . The roots of the equation are r = −2 , 1 . Hence the general
solution is y = c1e−2t
+ c2et
. Its derivative is y = −2c1e−2t
+ c2et
. Based on the
first condition, y(0) = 1 , we require that c1 + c2 = 1 . In order to satisfy y (0) = 1 ,
we find that −2c1 + c2 = 1 . Solving for the constants, c1 = 0 and c2 = 1 . Hence
the specific solution is y(t) = et
. It clearly increases without bound as t → ∞.
Elementary Differential Equations 11th Edition Boyce Solutions Manual
Full Download: https://alibabadownload.com/product/elementary-differential-equations-11th-edition-boyce-solutions-manual/
This sample only, Download all chapters at: AlibabaDownload.com
54 Chapter 3. Second-Order Linear Equations
9. The characteristic equation is r2
+ 3r = 0 , with roots r = −3 , 0 . Therefore
the general solution is y = c1 + c2e−3t
, with derivative y = −3 c2e−3t
. In order
to satisfy the initial conditions, we find that c1 + c2 = −2 , and −3 c2 = 3 . Hence
the specific solution is y(t) = −1 − e−3t
. This converges to −1 as t → ∞.
10. The characteristic equation is 2r2
+ r − 4 = 0 , with roots r = (−1 ±
√
33 )/4.
The general solution is y = c1e(−1−
√
33)t/4
+ c2e(−1+
√
33)t/4
, with derivative
y =
−1 −
√
33
4
c1e(−1−
√
33)t/4
+
−1 +
√
33
4
c2e(−1+
√
33)t/4
.
In order to satisfy the initial conditions, we require that
c1 + c2 = 0 and
−1 −
√
33
4
c1 +
−1 +
√
33
4
c2 = 1.
Solving for the coefficients, c1 = −2/
√
33 and c2 = 2/
√
33 . The specific solution
is
y(t) = −2 e(−1−
√
33)t/4
− e(−1+
√
33)t/4
/
√
33 .
It clearly increases without bound as t → ∞.
3.1 55
12. The characteristic equation is 4r2
− 1 = 0 , with roots r = ±1/2 . Therefore the
general solution is y = c1e−t/2
+ c2et/2
. Since the initial conditions are specified
at t = −2 , is more convenient to write y = d1e−(t+2)/2
+ d2e(t+2)/2
. The derivative
is given by y = − d1e−(t+2)/2
/2 + d2e(t+2)/2
/2 . In order to satisfy the initial
conditions, we find that d1 + d2 = 1 , and −d1/2 + d2/2 = −1 . Solving for the
coefficients, d1 = 3/2 , and d2 = −1/2 . The specific solution is
y(t) =
3
2
e−(t+2)/2
−
1
2
e(t+2)/2
=
3
2e
e−t/2
−
e
2
et/2
.
It clearly decreases without bound as t → ∞.
15. The characteristic equation is 2r2
− 3r + 1 = 0 , with roots r = 1/2 , 1 . There-
fore the general solution is y = c1et/2
+ c2et
, with derivative y = c1et/2
/2 + c2et
.
In order to satisfy the initial conditions, we require c1 + c2 = 2 and c1/2 + c2 = 1/2 .
Solving for the coefficients, c1 = 3 , and c2 = −1 . The specific solution is y(t) =
3et/2
− et
. To find the stationary point, set y = 3et/2
/2 − et
= 0 . There is a
unique solution, with t1 = ln(9/4). The maximum value is then y(t1) = 9/4. To
find the x-intercept, solve the equation 3et/2
− et
= 0 . The solution is readily
found to be t2 = ln 9 ≈ 2.1972 .
17. The characteristic equation is r2
− (2α − 1)r + α(α − 1) = 0 . Examining the
coefficients, the roots are r = α , α − 1 . Hence the general solution of the differen-
tial equation is y(t) = c1eαt
+ c2e(α−1)t
. Assuming α ∈ R , all solutions will tend
to zero as long as α < 0 . On the other hand, all solutions will become unbounded
as long as α − 1 > 0 , that is, α > 1 .
56 Chapter 3. Second-Order Linear Equations
19.(a) The characteristic roots are r = −3 , −2 . The solution of the initial value
problem is y(t) = (6 + β)e−2t
− (4 + β)e−3t
.
(b) The maximum point has coordinates t0 = ln [(3(4 + β))/(2(6 + β))], y0 = 4(6 +
β)3
/(27(4 + β)2
) .
(c) y0 = 4(6 + β)3
/(27(4 + β)2
) ≥ 4 , as long as β ≥ 6 + 6
√
3 .
(d) limβ→∞ t0 = ln(3/2), limβ→∞ y0 = ∞ .
20.(a) Assuming that y is a constant, the differential equation reduces to c y = d .
Hence the only equilibrium solution is y = d/c .
(b) Setting y = Y + d/c , substitution into the differential equation results in the
equation a Y + b Y + c(Y + d/c ) = d. The equation satisfied by Y is a Y +
b Y + cY = 0.
3.2
1.
W(e2t
, e−3t/2
) =
e2t
e−3t/2
2e2t
−3
2 e−3t/2 = −
7
2
et/2
.
3.
W(e−2t
, t e−2t
) =
e−2t
te−2t
−2e−2t
(1 − 2t)e−2t = e−4t
.
4.
W(et
sin t , et
cos t) =
et
sin t et
cos t
et
(sin t + cos t) et
(cos t − sin t)
= −e2t
.
5.
W(cos2
θ , 1 + cos 2θ) =
cos2
θ 1 + cos 2θ
−2 sin θ cos θ −2 sin 2θ
= 0 .
6. Write the equation as y + (3/t)y = 1 . p(t) = 3/t is continuous for all t > 0 .
Since t0 > 0 , the IVP has a unique solution for all t > 0 .
7. Write the equation as y + (3/(t − 4))y + (4/t(t − 4))y = 2/t(t − 4) . The coef-
ficients are not continuous at t = 0 and t = 4 . Since t0 ∈ (0 , 4) , the largest interval
is 0 < t < 4 .
8. The coefficient 3 ln |t| is discontinuous at t = 0 . Since t0 > 0 , the largest interval
of existence is 0 < t < ∞ .
3.2 57
10. y1 = 2 . We see that t2
(2) − 2(t2
) = 0 . y2 = 2 t−3
, with t2
(y2 ) − 2(y2) = 0 .
Let y3 = c1t2
+ c2t−1
, then y3 = 2c1 + 2c2t−3
. It is evident that y3 is also a
solution.
13. No. Substituting y = sin(t2
) into the differential equation,
−4t2
sin(t2
) + 2 cos(t2
) + 2t cos(t2
)p(t) + sin(t2
)q(t) = 0 .
At t = 0, this equation becomes 2 = 0 (if we suppose that p(t) and q(t) are contin-
uous), which is impossible.
14. W(e2t
, g(t)) = e2t
g (t) − 2e2t
g(t) = 3e4t
. Dividing both sides by e2t
, we find
that g must satisfy the ODE g − 2g = 3e2t
. Hence g(t) = 3t e2t
+ c e2t
.
15. W(f , g) = fg − f g = t cos t − sin t , and W(u , v) = −4fg + 4f g . Hence
W(u , v) = −4t cos t + 4 sin t .
16. We compute
W(a1y1 + a2y2, b1y1 + b2y2) =
a1y1 + a2y2 b1y1 + b2y2
a1y1 + a2y2 b1y1 + b2y2
=
= (a1y1 + a2y2)(b1y1 + b2y2) − (b1y1 + b2y2)(a1y1 + a2y2) =
= a1b2(y1y2 − y1y2) − a2b1(y1y2 − y1y2) = (a1b2 − a2b1)W(y1, y2).
This now readily shows that y3 and y4 form a fundamental set of solutions if and
only if a1b2 − a2b1 = 0.
18. The general solution is y = c1e−3t
+ c2e−t
. W(e−3t
, e−t
) = 2e−4t
, and hence
the exponentials form a fundamental set of solutions. On the other hand, the fun-
damental solutions must also satisfy the conditions y1(1) = 1, y1(1) = 0; y2(1) = 0,
y2(1) = 1. For y1, the initial conditions require c1 + c2 = e, −3c1 − c2 = 0. The
coefficients are c1 = −e3
/2, c2 = 3e/2. For the solution y2, the initial conditions re-
quire c1 + c2 = 0, −3c1 − c2 = e. The coefficients are c1 = −e3
/2, c2 = e/2. Hence
the fundamental solutions are
y1 = −
1
2
e−3(t−1)
+
3
2
e−(t−1)
and y2 = −
1
2
e−3(t−1)
+
1
2
e−(t−1)
.
19. Yes. y1 = −4 cos 2t ; y2 = −4 sin 2t . W(cos 2t , sin 2t) = 2 .
20. Clearly, y1 = et
is a solution. y2 = (1 + t)et
, y2 = (2 + t)et
. Substitution into
the ODE results in (2 + t)et
− 2(1 + t)et
+ t et
= 0 . Furthermore, W(et
, tet
) = e2t
.
Hence the solutions form a fundamental set of solutions.
24. Writing the equation in standard form, we find that P(t) = sin t/ cos t . Hence
the Wronskian is W(t) = c e− (sin t/ cos t) dt
= c eln|cos t|
= c cos t , in which c is
some constant.
58 Chapter 3. Second-Order Linear Equations
25. Writing the equation in standard form, we find that P(x) = −2x/(1 − x2
).
The Wronskian is W(x) = c e− −2x/(1−x2
) dx
= c e− ln|1−x2
| = c/(1 − x2
), in which
c is some constant.
26. Rewrite the equation as p(t)y + p (t)y + q(t)y = 0 . After writing the equa-
tion in standard form, we have P(t) = p (t)/p(t) . Hence the Wronskian is
W(t) = c e− p (t)/p(t) dt
= c e− ln p(t)
= c/p(t) .
28. For the given differential equation, the Wronskian satisfies the first order dif-
ferential equation W + p(t)W = 0 . Given that W is constant, it is necessary that
p(t) ≡ 0 .
32. P = 1 , Q = x , R = 1 . We have P − Q + R = 0 . The equation is exact.
Note that (y ) + (xy) = 0 . Hence y + xy = c1 . This equation is linear, with
integrating factor µ = ex2
/2
. Therefore the general solution is
y(x) = c1e−x2
/2
x
x0
eu2
/2
du + c2e−x2
/2
.
34. P = x2
, Q = x, R = −1. We have P − Q + R = 0. The equation is exact.
Write the equation as (x2
y ) − (xy) = 0. After integration, we conclude that
x2
y − xy = c. Divide both sides of the differential equation by x2
. The resulting
equation is linear, with integrating factor µ = 1/x . Hence (y/x) = c x−3
. The
solution is y(t) = c1x−1
+ c2x .
36. P = x2
, Q = x , R = x2
− ν2
. Hence the coefficients are 2P − Q = 3x and
P − Q + R = x2
+ 1 − ν2
. The adjoint of the original differential equation is
given by x2
µ + 3x µ + (x2
+ 1 − ν2
)µ = 0 .
37. P = 1 , Q = 0 , R = −x . Hence the coefficients are given by 2P − Q = 0 and
P − Q + R = −x . Therefore the adjoint of the original equation is µ − x µ = 0 .
3.3
1. e2−3i
= e2
e−3i
= e2
(cos 3 − i sin 3).
2. eiπ
= cos π + i sin π = −1 .
3. e2−(π/2) i
= e2
(cos(π/2) − i sin(π/2)) = −e2
i .
6. The characteristic equation is r2
− 2r + 6 = 0 , with roots r = 1 ± i
√
5 . Hence
the general solution is y = c1et
cos
√
5 t + c2 et
sin
√
5 t.
7. The characteristic equation is r2
+ 2r + 2 = 0 , with roots r = −1 ± i. Hence
the general solution is y = c1e−t
cos t + c2 e−t
sin t.
3.3 59
9. The characteristic equation is r2
+ 2r + 1.25 = 0, with roots r = −1 ± i/2.
Hence the general solution is y = c1e−t
cos(t/2) + c2 e−t
sin(t/2).
11. The characteristic equation is r2
+ 4r + 6.25 = 0 , with roots r = −2 ± (3/2) i.
Hence the general solution is y = c1e−2t
cos(3t/2) + c2 e−2t
sin(3t/2).
12. The characteristic equation is r2
+ 4 = 0 , with roots r = ± 2i. Hence the
general solution is y = c1 cos 2t + c2 sin 2t . Now y = −2c1 sin 2t + 2c2 cos 2t .
Based on the first condition, y(0) = 0 , we require that c1 = 0 . In order to satisfy
the condition y (0) = 1 , we find that 2c2 = 1 . The constants are c1 = 0 and
c2 = 1/2 . Hence the specific solution is y(t) = sin 2t /2. The solution is periodic.
13. The characteristic equation is r2
− 2r + 5 = 0 , with roots r = 1 ± 2i. Hence
the general solution is y = c1et
cos 2t + c2 et
sin 2t . Based on the initial condition
y(π/2) = 0 , we require that c1 = 0 . It follows that y = c2 et
sin 2t , and so the
first derivative is y = c2 et
sin 2t + 2c2 et
cos 2t . In order to satisfy the condition
y (π/2) = 2, we find that −2eπ/2
c2 = 2 . Hence we have c2 = −e−π/2
. There-
fore the specific solution is y(t) = −et−π/2
sin 2t . The solution oscillates with an
exponentially growing amplitude.
14. The characteristic equation is r2
+ 1 = 0 , with roots r = ± i. Hence the gen-
eral solution is y = c1 cos t + c2 sin t . Its derivative is y = −c1 sin t + c2 cos t .
Based on the first condition, y(π/3) = 2 , we require that c1 +
√
3 c2 = 4 . In or-
der to satisfy the condition y (π/3) = −4 , we find that −
√
3 c1 + c2 = −8 . Solving
60 Chapter 3. Second-Order Linear Equations
these for the constants, c1 = 1 + 2
√
3 and c2 =
√
3 − 2 . Hence the specific solution
is a steady oscillation, given by y(t) = (1 + 2
√
3) cos t + (
√
3 − 2) sin t .
17.(a) The characteristic equation is 5r2
+ 2r + 7 = 0 , with roots r = −(1 ± i
√
34)/5.
The solution is u = c1e−t/5
cos
√
34t/5 + c2e−t/5
sin
√
34t/5 . Invoking the given
initial conditions, we obtain the equations for the coefficients : c1 = 2 , −2 +
√
34 c2 =
5 . That is, c1 = 2 , c2 = 7/
√
34 . Hence the specific solution is
u(t) = 2e−t/5
cos
√
34
5
t +
7
√
34
e−t/5
sin
√
34
5
t .
(b) Based on the graph of u(t), T is in the interval 14 < t < 16 . A numerical
solution on that interval yields T ≈ 14.5115 .
19. Direct calculation gives the result. On the other hand, it can be shown that
W(f g , f h) = f2
W(g , h). Hence W(eλt
cos µt , eλt
sin µt) = e2λt
W(cos µt , sin µt) =
e2λt
[cos µt(sin µt) − (cos µt) sin µt] = µ e2λt
.
20.(a) Clearly, y1 and y2 are solutions. Also, W(cos t, sin t) = cos2
t + sin2
t = 1.
(b) y = i eit
, y = i2
eit
= −eit
. Evidently, y is a solution and so y = c1y1 + c2y2.
(c) Setting t = 0 , 1 = c1 cos 0 + c2 sin 0 , and c1 = 1.
3.3 61
(d) Differentiating, i eit
= c2 cos t . Setting t = 0 , i = c2 cos 0 and hence c2 = i .
Therefore eit
= cos t + i sin t .
21. Euler’s formula is eit
= cos t + i sin t . It follows that e−it
= cos t − i sin t .
Adding these equation, eit
+ e−it
= 2 cos t . Subtracting the two equations results
in eit
− e−it
= 2i sin t .
22. Let r1 = λ1 + iµ1 , and r2 = λ2 + iµ2 . Then
e(r1+r2)t
= e(λ1+λ2)t+i(µ1+µ2)t
= e(λ1+λ2)t
[cos(µ1 + µ2)t + i sin(µ1 + µ2)t] =
= e(λ1+λ2)t
[(cos µ1t + i sin µ1t)(cos µ2t + i sin µ2t)] =
= eλ1t
(cos µ1t + i sin µ1t) · eλ2t
(cos µ1t + i sin µ1t) = er1t
er2t
.
Hence e(r1+r2)t
= er1t
er2t
.
23. Clearly, u = λeλt
cos µt − µeλt
sin µt = eλt
(λ cos µt − µ sin µt) and then u =
λeλt
(λ cos µt − µ sin µt) + eλt
(−λµ sin µt − µ2
cos µt). Plugging these into the dif-
ferential equation, dividing by eλt
= 0 and arranging the sine and cosine terms we
obtain that the identity to prove is
(a(λ2
− µ2
) + bλ + c) cos µt + (−2λµa − bµ) sin µt = 0.
We know that λ ± iµ solves the characteristic equation ar2
+ br + c = 0, so a(λ −
iµ)2
+ b(λ − iµ) + c = a(λ2
− µ2
) + bλ + c + i(−2λµa − µb) = 0. If this complex
number is zero, then both the real and imaginary parts of it are zero, but those
are the coefficients of cos µt and sin µt in the above identity, which proves that
au + bu + cu = 0. The solution for v is analogous.
26. The equation transforms into y + y = 0. The characteristic roots are r = ±i.
The solution is y = c1 cos(x) + c2 sin(x) = c1 cos(ln t) + c2 sin(ln t).
28. The equation transforms into y − 5y − 6y = 0. The characteristic roots are
r = −1, 6. The solution is y = c1e−x
+ c2e6x
= c1e− ln t
+ c2e6 ln t
= c1/t + c2t6
.
29. The equation transforms into y − 5y + 6y = 0. The characteristic roots are
r = 2, 3. The solution is y = c1e2x
+ c2e3x
= c1e2 ln t
+ c2e3 ln t
= c1t2
+ c2t3
.
30. The equation transforms into y + 2y − 3y = 0. The characteristic roots are
r = 1, −3. The solution is y = c1ex
+ c2e−3x
= c1eln t
+ c2e−3 ln t
= c1t + c2/t3
.
31. The equation transforms into y + 6y + 10y = 0. The characteristic roots are
r = −3 ± i. The solution is
y = c1e−3x
cos(x) + c2e−3x
sin(x) = c1
1
t3
cos(ln t) + c2
1
t3
sin(ln t).
32.(a) By the chain rule, y (x) = (dy/dx)x . In general, dz/dt = (dz/dx)(dx/dt).
Setting z = (dy/dt), we have
d2
y
dt2
=
dz
dx
dx
dt
=
d
dx
dy
dx
dx
dt
dx
dt
=
d2
y
dx2
dx
dt
dx
dt
+
dy
dx
d
dx
dx
dt
dx
dt
.
62 Chapter 3. Second-Order Linear Equations
However,
d
dx
dx
dt
dx
dt
=
d2
x
dt2
dt
dx
·
dx
dt
=
d2
x
dt2
.
Hence
d2
y
dt2
=
d2
y
dx2
dx
dt
2
+
dy
dx
d2
x
dt2
.
(b) Substituting the results in part (a) into the general differential equation, y +
p(t)y + q(t)y = 0 , we find that
d2
y
dx2
dx
dt
2
+
dy
dx
d2
x
dt2
+ p(t)
dy
dx
dx
dt
+ q(t)y = 0 .
Collecting the terms,
dx
dt
2
d2
y
dx2
+
d2
x
dt2
+ p(t)
dx
dt
dy
dx
+ q(t)y = 0 .
(c) Assuming (dx/dt)2
= k q(t) , and q(t) > 0 , we find that dx/dt = k q(t) , which
can be integrated. That is, x = u(t) = k q(t) dt = q(t) dt, since k = 1.
(d) Let k = 1 . It follows that d2
x/dt2
+ p(t)dx/dt = du/dt + p(t)u(t) = q /2
√
q +
p
√
q . Hence
d2
x
dt2
+ p(t)
dx
dt
/
dx
dt
2
=
q (t) + 2p(t)q(t)
2 [q(t)]
3/2
.
As long as dx/dt = 0 , the differential equation can be expressed as
d2
y
dx2
+
q (t) + 2p(t)q(t)
2 [q(t)]
3/2
dy
dx
+ y = 0.
(e) To find the analogue to the condition found in part d) for the case when q(t) < 0
we return to the conditions that make the coefficients on y, dy/dt and d2
y/dt2
proportional to each other. Since the coefficients on y and d2
y/dt2
are propor-
tional, (dx/dt)2
= αq(t), and we may take α = −1. Thus dx/dt = (−q(t))1/2
and
d2
y/dt2
= (−q /2)(−q)−1/2
. Since the coefficients on y and dy/dt are proportional,
there is a constant β with
β q =
d2
y
dt2
+ p(t)
dx
dt
=
−q
2
(−q)−1/2
+ p(−q)1/2
=
−q − 2pq
2(−q)1/2
and dividing each side of the equation by −q gives
−β =
−q − 2pq
2(−q)3/2
, or 2β =
q + 2pq
(−q)3/2
Thus the desired condition is that (q + 2pq)/(−q)3/2
must be a constant.
3.4 63
34. Note that p(t) = 3t and q(t) = t2
. We have x = t dt = t2
/2 . Furthermore,
q (t) + 2p(t)q(t)
2 [q(t)]
3/2
=
1 + 3t2
t2
.
The ratio is not constant, and therefore the equation cannot be transformed.
35. Note that p(t) = t − 1/t and q(t) = t2
. We have x = t dt = t2
/2 . Further-
more,
q (t) + 2p(t)q(t)
2 [q(t)]
3/2
= 1 .
The ratio is constant, and therefore the equation can be transformed. From Problem
32, the transformed equation is
d2
y
dx2
+
dy
dx
+ y = 0 .
Based on the methods in this section, the characteristic equation is r2
+ r + 1 = 0 ,
with roots r = (−1 ± i
√
3)/2. The general solution is y(x) = c1e−x/2
cos
√
3 x/2 +
c2 e−x/2
sin
√
3 x/2. Since x = t2
/2 , the solution in the original variable t is
y(t) = e−t2
/4
c1 cos (
√
3 t2
/4) + c2 sin (
√
3 t2
/4) .
36. Note that p(t) = t and q(t) = −e−t2
< 0 for −∞ < t < ∞. To proceed we must
confirm that (q + 2pq)/(−q)3/2
is a constant:
q + 2pq
(−q)3/2
=
2te−t2
+ 2t(−et2
)
(e−t2
)3/2
= 0.
Thus the differential equation can be transformed into an equation with constant
coefficients by letting x = u(t) = e−t2
/2
dt. Substituting x = u(t) in the differ-
ential equation found in part (b) of Problem 32 we obtain, after dividing by the
coefficient of d2
y/dx2
, the differential equation (d2
y/dx2
) − y = 0. Hence the gen-
eral solution of the original differential equation is y(t) = c1ex(t)
+ c2e−x(t)
, where
x(t) = e−t2
/2
dt.
3.4
2. The characteristic equation is 9r2
+ 6r + 1 = 0 , with the double root r = −1/3 .
The general solution is y(t) = c1e−t/3
+ c2t e−t/3
.
3. The characteristic equation is 4r2
− 4r − 3 = 0 , with roots r = −1/2 , 3/2 . The
general solution is y(t) = c1e−t/2
+ c2e3t/2
.
5. The characteristic equation is r2
− 6r + 9 = 0 , with the double root r = 3 . The
general solution is y(t) = c1e3t
+ c2t e3t
.
64 Chapter 3. Second-Order Linear Equations
6. The characteristic equation is 4r2
+ 17r + 4 = 0 , with roots r = −1/4 , −4 .
The general solution is y(t) = c1e−t/4
+ c2e−4t
.
7. The characteristic equation is 16r2
+ 24r + 9 = 0, with double root r = −3/4.
The general solution is y(t) = c1e−3t/4
+ c2t e−3t/4
.
8. The characteristic equation is 2r2
+ 2r + 1 = 0. We obtain the complex roots
r = (−1 ± i)/2. The general solution is y(t) = c1e−t/2
cos(t/2) + c2e−t/2
sin(t/2).
9. The characteristic equation is 9r2
− 12r + 4 = 0 , with the double root r = 2/3 .
The general solution is y(t) = c1e2t/3
+ c2t e2t/3
. Invoking the first initial condi-
tion, it follows that c1 = 2 . Now y (t) = (4/3 + c2)e2t/3
+ 2c2t e2t/3
/3 . Invoking
the second initial condition, 4/3 + c2 = −1 , or c2 = −7/3 . Hence we obtain the
solution y(t) = 2e2t/3
− (7/3)te2t/3
. Since the second term dominates for large t,
y(t) → −∞.
12. The characteristic roots are r1 = r2 = 1/2 . Hence the general solution is given
by y(t) = c1et/2
+ c2t et/2
. Invoking the initial conditions, we require that c1 = 2 ,
and that 1 + c2 = b. The specific solution is y(t) = 2et/2
+ (b − 1)t et/2
. Since the
second term dominates, the long-term solution depends on the sign of the coefficient
b − 1. The critical value is b = 1.
15.(a) The characteristic equation is r2
+ 2a r + a2
= (r + a)2
= 0 .
(b) With p(t) = 2a , Abel’s Formula becomes W(y1 , y2) = c e− 2a dt
= c e−2at
.
(c) y1(t) = e−at
is a solution. From part (b), with c = 1, e−at
y2(t) + a e−at
y2(t) =
e−2at
, which can be written as (eat
y2(t)) = 1, resulting in eat
y2(t) = t.
17.(a) If the characteristic equation ar2
+ br + c has equal roots r1, then ar2
1 +
br1 + c = a(r − r1)2
= 0. Then clearly L[ert
] = (ar2
+ br + c)ert
= a(r − r1)2
ert
.
This gives immediately that L[er1t
] = 0.
(b) Differentiating the identity in part (a) with respect to r we get (2ar + b)ert
+
(ar2
+ br + c)tert
= 2a(r − r1)ert
+ a(r − r1)2
tert
. Again, this gives L[ter1t
] = 0.
3.4 65
18. Set y2(t) = t2
v(t) . Substitution into the differential equation results in
t2
(t2
v + 4tv + 2v) − 4t(t2
v + 2tv) + 6t2
v = 0 .
After collecting terms, we end up with t4
v = 0 . Hence v(t) = c1 + c2t , and thus
y2(t) = c1t2
+ c2t3
. Setting c1 = 0 and c2 = 1 , we obtain y2(t) = t3
.
19. Set y2(t) = t v(t) . Substitution into the differential equation results in
t2
(tv + 2v ) + 2t(tv + v) − 2tv = 0 .
After collecting terms, we end up with t3
v + 4t2
v = 0 . This equation is linear
in the variable w = v . It follows that v (t) = c t−4
, and v(t) = c1t−3
+ c2 . Thus
y2(t) = c1t−2
+ c2t . Setting c1 = 1 and c2 = 0 , we obtain y2(t) = t−2
.
23. Direct substitution verifies that y1(t) = e−δx2
/2
is a solution of the differential
equation. Now set y2(x) = y1(x) v(x). Substitution of y2 into the equation results
in v − δxv = 0. This equation is linear in the variable w = v . An integrating
factor is µ = e−δx2
/2
. Rewrite the equation as [ e−δx2
/2
v ] = 0 , from which it
follows that v (x) = c1 eδx2
/2
. Integrating, we obtain
v(x) = c1
x
0
eδu2
/2
du + v(0).
Hence
y2(x) = c1e−δx2
/2
x
0
eδu2
/2
du + c2e−δx2
/2
.
Setting c2 = 0 , we obtain a second independent solution.
25. After writing the differential equation in standard form, we have p(t) = 3/t .
Based on Abel’s identity, W(y1, y2) = c1e− 3/t dt
= c1t−3
. As shown in Problem
24, two solutions of a second order linear equation satisfy (y2/y1) = W(y1, y2)/y2
1.
In the given problem, y1(t) = t−1
. Hence (t y2) = c1t−1
. Integrating both sides
of the equation, y2(t) = c1t−1
ln t + c2t−1
. Setting c1 = 1 and c2 = 0 we obtain
y2(t) = t−1
ln t.
27. Write the differential equation in standard form to find p(x) = 1/x. Based on
Abel’s identity, W(y1, y2) = c e− 1/x dx
= c x−1
. Two solutions of a second order
linear differential equation satisfy (y2/y1) = W(y1, y2)/y2
1 . In the given problem,
y1(x) = x−1/2
sin x . Hence
(
√
x
sin x
y2) = c
1
sin2
x
.
Integrating both sides of the equation, y2(x) = c1x−1/2
cos x + c2x−1/2
sin x. Set-
ting c1 = 1 and c2 = 0 , we obtain y2(x) = x−1/2
cos x .
29.(a) The characteristic equation is ar2
+ c = 0 . If a , c > 0 , then the roots are
r = ± i c/a . The general solution is
y(t) = c1 cos
c
a
t + c2 sin
c
a
t ,
66 Chapter 3. Second-Order Linear Equations
which is bounded.
(b) The characteristic equation is ar2
+ br = 0 . The roots are r = 0 , −b/a , and
hence the general solution is y(t) = c1 + c2e−bt/a
. Clearly, y(t) → c1 . With the
given initial conditions, c1 = y0 + (a/b)y0.
30. Note that 2 cos t sin t = sin 2t. Then 1 − k cos t sin t = 1 − (k/2) sin 2t . Now
if 0 < k < 2 , then (k/2) sin 2t < |sin 2t| and −(k/2) sin 2t > − |sin 2t|. Hence
1 − k cos t sin t = 1 −
k
2
sin 2t > 1 − |sin 2t| ≥ 0 .
31. The equation transforms into y − 4y + 4y = 0. We obtain a double root r = 2.
The solution is y = c1e2x
+ c2xe2x
= c1e2 ln t
+ c2 ln te2 ln t
= c1t2
+ c2t2
ln t.
33. The equation transforms into y + 2y + y = 0. We get a double root r = −1.
The solution is y = c1e−x
+ c2xe−x
= c1e− ln t
+ c2 ln te− ln t
= c1t−1
+ c2t−1
ln t.
34. The equation transforms into y − 3y + 9y/4 = 0. We obtain the double
root r = 3/2. The solution is y = c1e3x/2
+ c2xe3x/2
= c1e3 ln t/2
+ c2 ln te3 ln t/2
=
c1t3/2
+ c2t3/2
ln t.
3.5
2. The characteristic equation for the homogeneous problem is r2
− r − 2 = 0 , with
roots r = −1, 2. Hence yc(t) = c1e−t
+ c2e2t
. Set Y = At2
+ Bt + C. Substitution
into the given differential equation, and comparing the coefficients, results in the
system of equations −2A = 4, −2A − 2B = −2 and 2A − B − 2C = 0. Hence Y =
−2t2
+ 3t − 7/2. The general solution is y(t) = yc(t) + Y .
3. The characteristic equation for the homogeneous problem is r2
+ r − 6 = 0 , with
roots r = −3, 2. Hence yc(t) = c1e−3t
+ c2e2t
. Set Y = Ae3t
+ Be−2t
. Substitu-
tion into the given differential equation, and comparing the coefficients, results in
the system of equations 6A = 12 and −4B = 12. Hence Y = 2e3t
− 3e−2t
. The
general solution is y(t) = yc(t) + Y .
4. The characteristic equation for the homogeneous problem is r2
− 2r − 3 = 0 ,
with roots r = −1 , 3 . Hence yc(t) = c1e−t
+ c2e3t
. Note that the assignment
Y = Ate−t
is not sufficient to match the coefficients. Try Y = Ate−t
+ Bt2
e−t
.
Substitution into the differential equation, and comparing the coefficients, results
in the system of equations −4A + 2B = 0 and −8B = −3 . This implies that
Y = (3/16)te−t
+ (3/8)t2
e−t
. The general solution is y(t) = yc(t) + Y .
8. The characteristic equation for the homogeneous problem is r2
+ ω2
0 = 0 , with
complex roots r = ± ω0 i. Hence yc(t) = c1 cos ω0 t + c2 sin ω0 t . Since ω = ω0 ,
set Y = A cos ωt + B sin ωt . Substitution into the ODE and comparing the co-
efficients results in the system of equations (ω2
0 − ω2
)A = 1 and (ω2
0 − ω2
)B = 0 .
3.5 67
Hence
Y =
1
ω2
0 − ω2
cos ωt .
The general solution is y(t) = yc(t) + Y .
9. From Problem 8, yc(t) is known. Since cos ω0 t is a solution of the homogeneous
problem, set Y = At cos ω0 t + Bt sin ω0 t . Substitution into the given ODE and
comparing the coefficients results in A = 0 and B = 1/2ω0 . Hence the general
solution is y(t) = c1 cos ω0 t + c2 sin ω0 t + t sin ω0 t/(2ω0 ).
12. The characteristic equation for the homogeneous problem is r2
+ 4 = 0 , with
roots r = ± 2i. Hence yc(t) = c1 cos 2t + c2 sin 2t . Set Y1 = A + Bt + Ct2
. Com-
paring the coefficients of the respective terms, we find that A = −1/8 , B = 0 ,
C = 1/4 . Now set Y2 = D et
, and obtain D = 3/5 . Hence the general solution is
y(t) = c1 cos 2t + c2 sin 2t − 1/8 + t2
/4 + 3 et
/5. Invoking the initial conditions, we
require that 19/40 + c1 = 0 and 3/5 + 2c2 = 2 . Hence c1 = −19/40 and c2 = 7/10 .
13. The characteristic equation for the homogeneous problem is r2
− 2r + 1 = 0 ,
with a double root r = 1 . Hence yc(t) = c1et
+ c2t et
. Consider g1(t) = t et
. Note
that g1 is a solution of the homogeneous problem. Set Y1 = At2
et
+ Bt3
et
(the first
term is not sufficient for a match). Upon substitution, we obtain Y1 = t3
et
/6 . By
inspection, Y2 = 4 . Hence the general solution is y(t) = c1et
+ c2t et
+ t3
et
/6 + 4 .
Invoking the initial conditions, we require that c1 + 4 = 1 and c1 + c2 = 1 . Hence
c1 = −3 and c2 = 4 .
14. The characteristic equation for the homogeneous problem is r2
+ 4 = 0 , with
roots r = ±2i. Hence yc(t) = c1 cos 2t + c2 sin 2t . Since the function sin 2t is
a solution of the homogeneous problem, set Y = At cos 2t + Bt sin 2t . Upon
substitution, we obtain Y = −3t cos 2t /4. Hence the general solution is y(t) =
c1 cos 2t + c2 sin 2t − 3t cos 2t /4. Invoking the initial conditions, we require that
c1 = 2 and 2c2 − (3/4) = −1 . Hence c1 = 2 and c2 = −1/8 .
15. The characteristic equation for the homogeneous problem is r2
+ 2r + 5 =
0 , with complex roots r = −1 ± 2i. Hence yc(t) = c1e−t
cos 2t + c2e−t
sin 2t .
Based on the form of g(t), set Y = At e−t
cos 2t + Bt e−t
sin 2t . After compar-
ing coefficients, we obtain Y = t e−t
sin 2t . Hence the general solution is y(t) =
c1e−t
cos 2t + c2e−t
sin 2t + t e−t
sin 2t. Invoking the initial conditions, we require
that c1 = 1 and −c1 + 2c2 = 0 . Hence c1 = 1 and c2 = 1/2 .
17.(a) The characteristic equation for the homogeneous problem is r2
− 5r + 6 = 0 ,
with roots r = 2 , 3. Hence yc(t) = c1e2t
+ c2e3t
. Consider g1(t) = e2t
(3t + 4) sin t ,
and g2(t) = et
cos 2t. Based on the form of these functions on the right hand side
of the ODE, set Y2(t) = et
(A1 cos 2t + A2 sin 2t) and Y1(t) = (B1 + B2t)e2t
sin t +
(C1 + C2t)e2t
cos t.
(b) Substitution into the equation and comparing the coefficients results in
Y (t) = −
1
20
(et
cos 2t + 3et
sin 2t) +
3
2
te2t
(cos t − sin t) + e2t
(
1
2
cos t − 5 sin t).
68 Chapter 3. Second-Order Linear Equations
19.(a) The homogeneous solution is yc(t) = c1 cos 2t + c2 sin 2t. Since cos 2t and
sin 2t are both solutions of the homogeneous equation, set
Y (t) = t(A0 + A1t + A2t2
) cos 2t + t(B0 + B1t + B2t2
) sin 2t .
(b) Substitution into the equation and comparing the coefficients results in
Y (t) = (
13
32
t −
1
12
t3
) cos 2t +
1
16
(28t + 13t2
) sin 2t .
20.(a) The homogeneous solution is yc(t) = c1e−t
+ c2te−2t
. None of the functions
on the right hand side are solutions of the homogenous equation. In order to include
all possible combinations of the derivatives, consider
Y (t) = et
(A0 + A1t + A2t2
) cos 2t + et
(B0 + B1t + B2t2
) sin 2t +
+ e−t
(C1 cos t + C2 sin t) + Det
.
(b) Substitution into the differential equation and comparing the coefficients results
in
Y (t) = et
(A0 + A1t + A2t2
) cos 2t + et
(B0 + B1t + B2t2
) sin 2t
+ e−t
(−
3
2
cos t +
3
2
sin t) + 2et
/3,
in which A0 = −4105/35152, A1 = 73/676, A2 = −5/52, B0 = −1233/35152, B1 =
10/169, B2 = 1/52.
21.(a) The homogeneous solution is yc(t) = c1e−t
cos 2t + c2e−t
sin 2t. None of the
terms on the right hand side are solutions of the homogenous equation. In order to
include the appropriate combinations of derivatives, consider
Y (t) = e−t
(A1t + A2t2
) cos 2t + e−t
(B1t + B2t2
) sin 2t +
+ e−2t
(C0 + C1t) cos 2t + e−2t
(D0 + D1t) sin 2t.
(b) Substitution into the differential equation and comparing the coefficients results
in
Y (t) =
3
16
te−t
cos 2t +
3
8
t2
e−t
sin 2t
−
1
25
e−2t
(7 + 10t) cos 2t +
1
25
e−2t
(1 + 5t) sin 2t .
23. The homogeneous solution is yc(t) = c1 cos λt + c2 sin λt. Since the differential
operator does not contain a first derivative (and λ = mπ), we can set
Y (t) =
N
m=1
Cm sin mπt .
3.5 69
Substitution into the differential equation yields
−
N
m=1
m2
π2
Cm sin mπt + λ2
N
m=1
Cm sin mπt =
N
m=1
am sin mπt .
Equating coefficients of the individual terms, we obtain
Cm =
am
λ2 − m2π2
, m = 1, 2 . . . N.
25. Since a , b, c > 0 , the roots of the characteristic equation have negative real
parts. That is, r = α ± β i , where α < 0 . Hence the homogeneous solution is
yc(t) = c1eαt
cos βt + c2eαt
sin βt .
If g(t) = d , then the general solution is
y(t) = d/c + c1eαt
cos βt + c2eαt
sin βt .
Since α < 0 , y(t) → d/c as t → ∞ . If c = 0, then the characteristic roots are
r = 0 and r = −b/a . The ODE becomes ay + by = d . Integrating both sides, we
find that ay + by = d t + c1. The general solution can be expressed as
y(t) = d t/b + c1 + c2e−bt/a
.
In this case, the solution grows without bound. If b = 0, also, then the differential
equation can be written as y = d/a , which has general solution y(t) = d t2
/2a +
c1 + c2 . Hence the assertion is true only if the coefficients are positive.
27.(a) Since D is a linear operator, D2
y + bDy + cy = D2
y − (r1 + r2)Dy + r1r2y =
D2
y − r2Dy − r1Dy + r1r2y = D(Dy − r2y) − r1(Dy − r2y) = (D − r1)(D − r2)y.
(b) Let u = (D − r2)y . Then the ODE (i) can be written as (D − r1)u = g(t),
that is, u − r1u = g(t). The latter is a linear first order equation in u . Its general
solution is
u(t) = er1t
t
t0
e−r1τ
g(τ)dτ + c1er1t
.
From above, we have y − r2y = u(t). This equation is also a first order ODE.
Hence the general solution of the original second order equation is
y(t) = er2t
t
t0
e−r2τ
u(τ)dτ + c2er2t
.
Note that the solution y(t) contains two arbitrary constants.
29. We have (D2
+ 2D + 1)y = (D + 1)(D + 1)y . Let u = (D + 1)y , and consider
the ODE u + u = 2e−t
. The general solution is u(t) = 2t e−t
+ c e−t
. We therefore
have the first order equation u + u = 2t e−t
+ c1e−t
. The general solution of the
latter differential equation is
y(t) = e−t
t
t0
[2τ + c1 ] dτ + c2e−t
= e−t
(t2
+ c1t + c2).
70 Chapter 3. Second-Order Linear Equations
30. We have (D2
+ 2D)y = D(D + 2)y . Let u = (D + 2)y , and consider the equa-
tion u = 3 + 4 sin 2t . Direct integration results in u(t) = 3t − 2 cos 2t + c . The
problem is reduced to solving the ODE y + 2y = 3t − 2 cos 2t + c . The general
solution of this first order differential equation is
y(t) = e−2t
t
t0
e2τ
[3τ − 2 cos 2τ + c ] dτ + c2e−2t
=
=
3
2
t −
1
2
(cos 2t + sin 2t) + c1 + c2e−2t
.
3.6
1. The solution of the homogeneous equation is yc(t) = c1e2t
+ c2e3t
. The functions
y1(t) = e2t
and y2(t) = e3t
form a fundamental set of solutions. The Wronskian
of these functions is W(y1, y2) = e5t
. Using the method of variation of parameters,
the particular solution is given by Y (t) = u1(t) y1(t) + u2(t) y2(t), in which
u1(t) = −
e3t
(2et
)
W(t)
dt = 2e−t
and u2(t) =
e2t
(2et
)
W(t)
dt = −e−2t
.
Hence the particular solution is Y (t) = 2et
− et
= et
.
3. The functions y1(t) = et/2
and y2(t) = tet/2
form a fundamental set of solutions.
The Wronskian of these functions is W(y1, y2) = et
. First write the equation in
standard form, so that g(t) = 4et/2
. Using the method of variation of parameters,
the particular solution is given by Y (t) = u1(t) y1(t) + u2(t) y2(t), in which
u1(t) = −
tet/2
(4et/2
)
W(t)
dt = −2t2
and u2(t) =
et/2
(4et/2
)
W(t)
dt = 4t.
Hence the particular solution is Y (t) = −2t2
et/2
+ 4t2
et/2
= 2t2
et/2
.
5. The solution of the homogeneous equation is yc(t) = c1 cos 3t + c2 sin 3t. The
two functions y1(t) = cos 3t and y2(t) = sin 3t form a fundamental set of solu-
tions, with W(y1, y2) = 3 . The particular solution is given by Y (t) = u1(t) y1(t) +
u2(t) y2(t), in which
u1(t) = −
sin 3t(9 sec2
3t)
W(t)
dt = − csc 3t
u2(t) =
cos 3t(9 sec2
3t)
W(t)
dt = ln(sec 3t + tan 3t),
since 0 < t < π/6. Hence Y (t) = −1 + (sin 3t) ln(sec 3t + tan 3t). The general so-
lution is given by
y(t) = c1 cos 3t + c2 sin 3t + (sin 3t) ln(sec 3t + tan 3t) − 1 .
3.6 71
6. The functions y1(t) = e−2t
and y2(t) = te−2t
form a fundamental set of so-
lutions. The Wronskian of these functions is W(y1, y2) = e−4t
. The particular
solution is given by Y (t) = u1(t) y1(t) + u2(t) y2(t), in which
u1(t) = −
te−2t
(t−2
e−2t
)
W(t)
dt = − ln t and u2(t) =
e−2t
(t−2
e−2t
)
W(t)
dt = −1/t.
Hence the particular solution is Y (t) = −e−2t
ln t − e−2t
. Since the second term is
a solution of the homogeneous equation, the general solution is given by
y(t) = c1e−2t
+ c2te−2t
− e−2t
ln t.
7. The functions y1(t) = cos (t/2) and y2(t) = sin(t/2) form a fundamental set of
solutions. The Wronskian of these functions is W(y1, y2) = 1/2 . First write the
ODE in standard form, so that g(t) = sec(t/2)/2. The particular solution is given
by Y (t) = u1(t) y1(t) + u2(t) y2(t), in which
u1(t) = −
cos (t/2) [sec(t/2)]
2W(t)
dt = 2 ln(cos (t/2))
u2(t) =
sin(t/2) [sec(t/2)]
2W(t)
dt = t.
The particular solution is Y (t) = 2 cos(t/2) ln(cos (t/2)) + t sin(t/2). The general
solution is given by
y(t) = c1 cos (t/2) + c2 sin(t/2) + 2 cos(t/2) ln(cos (t/2)) + t sin(t/2).
8. The solution of the homogeneous equation is yc(t) = c1et
+ c2tet
. The functions
y1(t) = et
and y2(t) = tet
form a fundamental set of solutions, with W(y1, y2) =
e2t
. The particular solution is given by Y (t) = u1(t) y1(t) + u2(t) y2(t), in which
u1(t) = −
tet
(et
)
W(t)(1 + t2)
dt = −
1
2
ln(1 + t2
)
u2(t) =
et
(et
)
W(t)(1 + t2)
dt = arctan t.
The particular solution is Y (t) = −(1/2)et
ln(1 + t2
) + tet
arctan(t). Hence the
general solution is given by
y(t) = c1et
+ c2tet
−
1
2
et
ln(1 + t2
) + tet
arctan(t).
10. Note first that p(t) = 0 , q(t) = −2/t2
and g(t) = (3t2
− 1)/t2
. The functions
y1(t) and y2(t) are solutions of the homogeneous equation, verified by substitution.
The Wronskian of these two functions is W(y1,y2) = −3 . Using the method of
variation of parameters, the particular solution is Y (t) = u1(t) y1(t) + u2(t) y2(t),
in which
u1(t) = −
t−1
(3t2
− 1)
t2 W(t)
dt = t−2
/6 + ln t
72 Chapter 3. Second-Order Linear Equations
u2(t) =
t2
(3t2
− 1)
t2 W(t)
dt = −t3
/3 + t/3.
Therefore Y (t) = 1/6 + t2
ln t − t2
/3 + 1/3 .
12. Observe that g(t) = t e2t
. The functions y1(t) and y2(t) are a fundamental set
of solutions. The Wronskian of these two functions is W(y1,y2) = t et
. Using the
method of variation of parameters, the particular solution is Y (t) = u1(t) y1(t) +
u2(t) y2(t), in which
u1(t) = −
et
(t e2t
)
W(t)
dt = −e2t
/2 and u2(t) =
(1 + t)(t e2t
)
W(t)
dt = t et
.
Therefore Y (t) = −(1 + t)e2t
/2 + t e2t
= −e2t
/2 + t e2t
/2 .
13. Note that g(x) = ln x . The functions y1(x) = x2
and y2(x) = x2
ln x are solu-
tions of the homogeneous equation, as verified by substitution. The Wronskian of
the solutions is W(y1,y2) = x3
. Using the method of variation of parameters, the
particular solution is Y (x) = u1(x) y1(x) + u2(x) y2(x), in which
u1(x) = −
x2
ln x(ln x)
W(x)
dx = −(ln x)3
/3
u2(x) =
x2
(ln x)
W(x)
dx = (ln x)2
/2.
Therefore Y (x) = −x2
(ln x)3
/3 + x2
(ln x)3
/2 = x2
(ln x)3
/6 .
15. First write the equation in standard form. The forcing function becomes
g(x)/x2
. The functions y1(x) = x−1/2
sin x and y2(x) = x−1/2
cos x are a fun-
damental set of solutions. The Wronskian of the solutions is W(y1,y2) = −1/x .
Using the method of variation of parameters, the particular solution is Y (x) =
u1(x) y1(x) + u2(x) y2(x), in which
u1(x) =
x
x0
cos τ (g(τ))
τ
√
τ
dτ and u2(x) = −
x
x0
sin τ (g(τ))
τ
√
τ
dτ.
Therefore
Y (x) =
sin x
√
x
x
x0
cos τ (g(τ))
τ
√
τ
dt −
cos x
√
x
x
x0
sin τ (g(τ))
τ
√
τ
dτ =
=
1
√
x
x
x0
sin(x − τ) g(τ)
τ
√
τ
dτ .
16. Eq.(28) is
Y (t) = −y1(t)
t
t0
y2(s)g(s)
W(y1, y2)(s)
ds + y2(t)
t
t0
y1(s)g(s)
W(y1, y2)(s)
ds,
where t0 is now considered the initial point. Bringing the terms y1(t) and y2(t)
inside the integrals and using the fact that W(y1, y2)(s) = y1(s)y2(s) − y1(s)y2(s),
the desired result holds. To show that Y (t) satisfies L[y] = g(t) we must take
the derivative using Leibniz’s rule, which says that if y(t) =
t
t0
G(t, s) ds, then
3.6 73
Y (t) = G(t, t) +
t
t0
Gt(t, s) ds. Letting G(t, s) be the above integrand, we have
that G(t, t) = 0 and
∂G
∂t
=
y1(s)y2(t) − y1(t)y2(s)
W(y1, y2)(s)
g(s).
Likewise,
Y =
∂G(t, t)
∂t
+
t
t0
∂2
G
∂t2
(t, s) ds = g(t) +
t
t0
y1(s)y2 (t) − y1 (t)y2(s)
W(y1, y2)(s)
g(s) ds.
Since y1 and y2 are solutions of L[y] = 0, we have L[Y ] = g(t) since all the terms
involving the integral will add to zero. Clearly y(t0) = 0 and y (t0) = 0.
17. Let y1(t) and y2(t) be a fundamental set of solutions, and W(t) = W(y1, y2) be
the corresponding Wronskian. Any solution, u(t), of the homogeneous equation is
a linear combination u(t) = α1y1(t) + α2y2(t). Invoking the initial conditions, we
require that
y0 = α1 y1(t0) + α2 y2(t0)
y0 = α1 y1(t0) + α2 y2(t0)
Note that this system of equations has a unique solution, since W(t0) = 0. Now
consider the nonhomogeneous problem, L [v] = g(t), with homogeneous initial con-
ditions. Using the method of variation of parameters, the particular solution is
given by
Y (t) = −y1(t)
t
t0
y2(s) g(s)
W(s)
ds + y2(t)
t
t0
y1(s) g(s)
W(s)
ds .
The general solution of the IVP (iii) is
v(t) = β1y1(t) + β2y2(t) + Y (t) = β1y1(t) + β2y2(t) + y1(t)u1(t) + y2(t)u2(t)
in which u1 and u2 are defined above. Invoking the initial conditions, we require
that
0 = β1y1(t0) + β2y2(t0) + Y (t0)
0 = β1y1(t0) + β2y2(t0) + Y (t0)
Based on the definition of u1 and u2 , Y (t0) = 0 . Furthermore, since y1u1 + y2u2 =
0 , it follows that Y (t0) = 0 . Hence the only solution of the above system of equa-
tions is the trivial solution. Therefore v(t) = Y (t) . Now consider the function
y = u + v . Then L [y] = L [u + v] = L [u] + L [v] = g(t). That is, y(t) is a solution
of the nonhomogeneous problem. Further, y(t0) = u(t0) + v(t0) = y0 , and simi-
larly, y (t0) = y0 . By the uniqueness theorems, y(t) is the unique solution of the
initial value problem.
18.(a) A fundamental set of solutions is y1(t) = cos t and y2(t) = sin t . The Wron-
skian W(t) = y1y2 − y1y2 = 1 . By the result in Problem 17,
Y (t) =
t
t0
cos(s) sin(t) − cos(t) sin(s)
W(s)
g(s)ds
=
t
t0
[cos(s) sin(t) − cos(t) sin(s)] g(s)ds .
74 Chapter 3. Second-Order Linear Equations
Finally, we have cos(s) sin(t) − cos(t) sin(s) = sin(t − s).
(b) Using Problem 16 and part (a), the solution is
y(t) = y0 cos t + y0 sin t +
t
0
sin(t − s)g(s)ds .
19. A fundamental set of solutions is y1(t) = eat
and y2(t) = ebt
. The Wronskian
W(t) = y1y2 − y1y2 = (b − a)e(a+b)t
. By the result in Problem 17,
Y (t) =
t
t0
eas
ebt
− eat
ebs
W(s)
g(s)ds =
1
b − a
t
t0
eas
ebt
− eat
ebs
e(a+b)s
g(s)ds .
Hence the particular solution is
Y (t) =
1
b − a
t
t0
eb(t−s)
− ea(t−s)
g(s)ds .
21. A fundamental set of solutions is y1(t) = eat
and y2(t) = teat
. The Wronskian
W(t) = y1y2 − y1y2 = e2at
. By the result in Problem 17,
Y (t) =
t
t0
teas+at
− s eat+as
W(s)
g(s)ds =
t
t0
(t − s)eas+at
e2as
g(s)ds .
Hence the particular solution is
Y (t) =
t
t0
(t − s)ea(t−s)
g(s)ds .
22. The form of the kernel depends on the characteristic roots. If the roots are real
and distinct,
K(t − s) =
eb(t−s)
− ea(t−s)
b − a
.
If the roots are real and identical,
K(t − s) = (t − s)ea(t−s)
.
If the roots are complex conjugates,
K(t − s) =
eλ(t−s)
sin µ(t − s)
µ
.
23. Let y(t) = v(t)y1(t), in which y1(t) is a solution of the homogeneous equation.
Substitution into the given ODE results in
v y1 + 2v y1 + vy1 + p(t) [v y1 + vy1] + q(t)vy1 = g(t) .
By assumption, y1 + p(t)y1 + q(t)y1 = 0 , hence v(t) must be a solution of the ODE
v y1 + [2y1 + p(t)y1] v = g(t) .
Setting w = v , we also have w y1 + [2y1 + p(t)y1] w = g(t) .
3.7 75
25. First write the equation as y + 7t−1
y + 5t−2
y = t−1
. As shown in Problem
23, the function y(t) = t−1
v(t) is a solution of the given ODE as long as v is a
solution of
t−1
v + −2t−2
+ 7t−2
v = t−1
,
that is, v + 5t−1
v = 1 . This ODE is linear and first order in v . The integrating
factor is µ = t5
. The solution is v = t/6 + c t−5
. Direct integration now results in
v(t) = t2
/12 + c1t−4
+ c2 . Hence y(t) = t/12 + c1t−5
+ c2t−1
.
26. Write the equation as y − t−1
(1 + t)y + t−1
y = t e2t
. As shown in Problem
23, the function y(t) = (1 + t)v(t) is a solution of the given ODE as long as v is a
solution of
(1 + t) v + 2 − t−1
(1 + t)2
v = t e2t
,
that is,
v −
1 + t2
t(t + 1)
v =
t
t + 1
e2t
.
This equation is first order linear in v , with integrating factor µ = t−1
(1 + t)2
e−t
.
The solution is v = (t2
e2t
+ c1tet
)/(1 + t)2
. Integrating, we obtain v(t) = e2t
/2 −
e2t
/(t + 1) + c1et
/(t + 1) + c2 . Hence the solution of the original ODE is y(t) =
(t − 1)e2t
/2 + c1et
+ c2(t + 1) .
3.7
1. R cos δ = 3 and R sin δ = 4, so R =
√
25 = 5 and δ = arctan(4/3). We obtain
that u = 5 cos(2t − arctan(4/3)).
2. R cos δ = −2 and R sin δ = −3, so R =
√
13 and δ = π + arctan(3/2). We obtain
that u =
√
13 cos(πt − π − arctan(3/2)).
4. The spring constant is k = 3/(1/4) = 12 lb/ft . Mass m = 3/32 lb-s2
/ft . Since
there is no damping, the equation of motion is 3u /32 + 12u = 0, that is, u +
128u = 0 . The initial conditions are u(0) = −1/12 ft , u (0) = 2 ft/s. The general
solution is u(t) = A cos 8
√
2 t + B sin 8
√
2 t . Invoking the initial conditions, we
have
u(t) = −
1
12
cos 8
√
2 t +
1
4
√
2
sin 8
√
2 t .
R = 11/288 ft, δ = π − arctan(3/
√
2 ) rad, ω0 = 8
√
2 rad/s, T = π/(4
√
2 ) s.
6. The spring constant is k = 3/(.1) = 30 N/m . The damping coefficient is given as
γ = 3/5 N-s/m . Hence the equation of motion is 2u + 3u /5 + 30u = 0, that is,
u + 0.3u + 15u = 0 . The initial conditions are u(0) = 0.05 m and u (0) = 0.01
m/s. The general solution is u(t) = A cos µt + B sin µt , in which µ = 3.87008
rad/s . Invoking the initial conditions, we have u(t) = e−0.15t
(0.05 cos µt + 0.00452 sin µt).
Also, µ/ω0 = 3.87008/
√
15 ≈ 0.99925 .
76 Chapter 3. Second-Order Linear Equations
8. The frequency of the undamped motion is ω0 = 1 . The quasi frequency of the
damped motion is µ = 4 − γ2 /2. Setting µ = 2ω0 /3, we obtain γ = 2
√
5 /3.
9. The spring constant is k = mg/L . The equation of motion for an undamped
system is mu + mgu/L = 0. Hence the natural frequency of the system is ω0 =
g/L . The period is T = 2π/ω0 .
10. The general solution of the system is u(t) = A cos γ(t − t0) + B sin γ(t − t0) .
Invoking the initial conditions, we have u(t) = u0 cos γ(t − t0) + (u0/γ) sin γ(t −
t0). Clearly, the functions v = u0 cos γ(t − t0) and w = (u0/γ) sin γ(t − t0) satisfy
the given criteria.
11. Note that r sin( ω0t − θ) = r sin ω0t cos θ − r cos ω0t sin θ . Comparing the
given expressions, we have A = −r sin θ and B = r cos θ . That is, r = R =√
A2 + B2 , and tan θ = −A/B = −1/ tan δ . The latter relation is also tan θ +
cot δ = 1 .
12. The system is critically damped, when R = 2 L/C . Here R = 1000 ohms.
15.(a) Let u = Re−γt/2m
cos(µt − δ). Then attains a maximum when µtk − δ =
2kπ. Hence Td = tk+1 − tk = 2π/µ .
(b) u(tk)/u(tk+1) = e−γtk/2m
/e−γtk+1/2m
= e(γtk+1−γtk)/2m
. Hence u(tk)/u(tk+1) =
eγ(2π/µ)/2m
= eγ Td/2m
.
(c) ∆ = ln [u(tk)/u(tk+1)] = γ(2π/µ)/2m = πγ/µm .
16. The spring constant is k = 16/(1/4) = 64 lb/ft . Mass m = 1/2 lb-s2
/ft . The
damping coefficient is γ = 2 lb-s/ft . The quasi frequency is µ = 2
√
31 rad/s .
Hence ∆ = 2π/
√
31 ≈ 1.1285 .
18.(a) The characteristic equation is mr2
+ γr + k = 0 . Since γ2
< 4km , the roots
are r1,2 = (−γ ± i 4mk − γ2)/2m. The general solution is
u(t) = e−γt/2m
A cos
4mk − γ2
2m
t + B sin
4mk − γ2
2m
t .
Invoking the initial conditions, A = u0 and B = (2mv0 − γu0)/ 4mk − γ2 .
(b) We can write u(t) = R e−γt/2m
cos(µt − δ) , in which
R = u2
0 +
(2mv0 − γu0)2
4mk − γ2
and δ = arctan
(2mv0 − γu0)
u0 4mk − γ2
.
(c)
R = u2
0 +
(2mv0 − γu0)2
4mk − γ2
= 2
m(ku2
0 + γu0v0 + mv2
0)
4mk − γ2
=
a + bγ
4mk − γ2
.
3.7 77
It is evident that R increases (monotonically) without bound as γ → (2
√
mk)−
.
20.(a) The general solution is u(t) = A cos
√
2 t + B sin
√
2 t . Invoking the initial
conditions, we have u(t) =
√
2 sin
√
2 t .
(b)
(c)
The condition u (0) = 2 implies that u(t) initially increases. Hence the phase point
travels clockwise.
23. Based on Newton’s second law, with the positive direction to the right, F =
mu , where F = −ku − γu . Hence the equation of motion is mu + γu +
ku = 0 . The only difference in this problem is that the equilibrium position is
located at the unstretched configuration of the spring.
24.(a) The restoring force exerted by the spring is Fs = −(ku + u3
). The oppos-
ing viscous force is Fd = −γu . Based on Newton’s second law, with the pos-
itive direction to the right, Fs + Fd = mu . Hence the equation of motion is
mu + γu + ku + u3
= 0 .
(b) With the specified parameter values, the equation of motion is u + u = 0 .
The general solution of this ODE is u(t) = A cos t + B sin t . Invoking the initial
conditions, the specific solution is u(t) = sin t . Clearly, the amplitude is R = 1,
and the period of the motion is T = 2π .
78 Chapter 3. Second-Order Linear Equations
(c) Given = 0.1 , the equation of motion is u + u + 0.1 u3
= 0 . A solution of the
IVP can be generated numerically. We estimate A = 0.98 and T = 6.07.
(d) For = 0.2, A = 0.96 and T = 5.90. For = 0.3, A = 0.94 and T = 5.74.
(e) The amplitude and period both seem to decrease.
(f) For = −0.1, A = 1.03 and T = 6.55. For = −0.2, A = 1.06 and T = 6.90.
For = −0.3, A = 1.11 and T = 7.41. The amplitude and period both seem to
increase.
(a) ε = −0.1 (b) ε = −0.2 (c) ε = −0.3
3.8 79
3.8
1. We have sin(α ± β) = sin α cos β ± cos α sin β . Subtracting the two identities,
we obtain sin(α + β) − sin(α − β) = 2 cos α sin β . Setting α + β = 7t and α −
β = 6t, we get that α = 6.5t and β = 0.5t . This implies that sin 7t − sin 6t =
2 sin (t/2) cos (13t/2).
2. Consider the trigonometric identities cos(α ± β) = cos α cos β sin α sin β.
Adding the two identities, we get cos(α − β) + cos(α + β) = 2 cos α cos β . Com-
paring the expressions, set α + β = 2πt and α − β = πt. This means α = 3πt/2 and
β = πt/2 . Upon substitution, we have cos(πt) + cos(2πt) = 2 cos(3πt/2) cos(πt/2) .
3. Adding the two identities sin(α ± β) = sin α cos β ± cos α sin β , it follows
that sin(α − β) + sin(α + β) = 2 sin α cos β . Setting α + β = 4t and α − β = 3t,
we have α = 7t/2 and β = t/2 . Hence sin 3t + sin 4t = 2 sin(7t/2) cos(t/2) .
4. Using MKS units, the spring constant is k = 5(9.8)/0.1 = 490 N/m , and the
damping coefficient is γ = 2/0.04 = 50 N-s/m . The equation of motion is
5u + 50u + 490u = 10 sin(t/2) .
The initial conditions are u(0) = 0 m and u (0) = 0.03 m/s .
5.(a) The homogeneous solution is uc(t) = Ae−5t
cos
√
73 t + Be−5t
sin
√
73 t . Based
on the method of undetermined coefficients, the particular solution is
U(t) =
1
153281
[−160 cos(t/2) + 3128 sin(t/2)] .
Hence the general solution of the ODE is u(t) = uc(t) + U(t). Invoking the initial
conditions, we find that
A = 160/153281 and B = 383443
√
73 /1118951300 .
Hence the response is
u(t) =
1
153281
160 e−5t
cos
√
73 t +
383443
√
73
7300
e−5t
sin
√
73 t + U(t).
(b) uc(t) is the transient part and U(t) is the steady state part of the response.
80 Chapter 3. Second-Order Linear Equations
(c)
(d) The amplitude of the forced response is given by R = 2/∆, in which
∆ = 25(98 − ω2)2 + 2500 ω2 .
The maximum amplitude is attained when ∆ is a minimum. Hence the amplitude
is maximum at ω = 4
√
3 rad/s .
8. The equation of motion is 2u + u + 3u = 3 cos 3t − 2 sin 3t. Since the system
is damped, the steady state response is equal to the particular solution. Using
the method of undetermined coefficients, we obtain uss(t) = (sin 3t − cos 3t)/6.
Further, we find that R =
√
2 /6 and δ = arctan(−1) = 3π/4 . Hence we can write
uss(t) = (
√
2 /6) cos(3t − 3π/4).
9.(a) Plug in u(t) = R cos(ωt − δ) into the equation mu + γu + ku = F0 cos ωt,
then use trigonometric identities and compare the coefficients of cos ωt and sin ωt.
The result follows.
(b) First note that since R = F0/∆, Rk/F0 = k/∆ and that since Γ = γ2
/(mk),
(γ2
ω2
)/m2
= Γω2
0ω2
. Then using Eq.12,
Rk
F0
=
k
∆
=
mω2
0
m2(ω2
0 − ω2)2 + γ2ω2
=
mω2
0
m2(ω2
0 − ω2)2 + γ2ω2
=
ω2
0
(ω2
0 − ω2)2 + γ2ω2
m2
=
ω2
0
(ω2
0 − ω2)2 + Γω2
0ω2
=
1
ω2
0 −ω2
ω2
0
2
+ Γ
ω2
0 ω2
ω4
0
=
1
1 − ω2
ω2
0
2
+ Γω2
ω2
0
(c) The amplitude of the steady-state response is given by
R =
F0
m2(ω2
0 − ω2)2 + γ2 ω2
.
Since F0 is constant, the amplitude is maximum when the denominator of R is
minimum . Let z = ω2
, and consider the function f(z) = m2
(ω2
0 − z)2
+ γ2
z . Note
3.8 81
that f(z) is a quadratic, with minimum at z = ω2
0 − γ2
/2m2
. Hence the amplitude
R attains a maximum at ω2
max = ω2
0 − γ2
/2m2
. Furthermore, since ω2
0 = k/m ,
ω2
max = ω2
0 1 −
γ2
2km
.
(d) Substituting ω2
= ω2
max into the expression for the amplitude R gives the
maximum value for R:
Rmax =
F0
γ4/4m2 + γ2 (ω2
0 − γ2/2m2)
=
F0
ω2
0γ2 − γ4/4m2
=
F0
γω0 1 − γ2/4mk
.
To understand the approximation, note that
Rmax =
F0
γω0
1 −
γ2
4mk
−1/2
Recall that binomial theorem states that (1 + a)p
≈ 1 + pa when a is small. Ap-
plying this result with a = −γ2
/(4mk) and p = −1/2 gives that
Rmax =
F0
γω0
1 −
γ2
4mk
−1/2
≈
F0
γω0
1 + −
1
2
−
γ2
4mk
=
F0
γω0
1 +
γ2
8mk
13.(a) The homogeneous solution is uc(t) = A cos t + B sin t . Based on the method
of undetermined coefficients, the particular solution is
U(t) =
3
1 − ω2
cos ωt .
Hence the general solution of the ODE is u(t) = uc(t) + U(t). Invoking the initial
conditions, we find that A = 3/(ω2
− 1) and B = 0 . Hence the response is
u(t) =
3
1 − ω2
[ cos ωt − cos t ] .
(b)
(a) ω = 0.7 (b) ω = 0.8 (c) ω = 0.9
Note that
u(t) =
6
1 − ω2
sin
(1 − ω)t
2
sin
(ω + 1)t
2
.
82 Chapter 3. Second-Order Linear Equations
14.(a) The homogeneous solution is uc(t) = A cos t + B sin t . Based on the method
of undetermined coefficients, the particular solution is
U(t) =
3
1 − ω2
cos ωt .
Hence the general solution is u(t) = uc(t) + U(t). Invoking the initial conditions,
we find that A = (ω2
+ 2)/(ω2
− 1) and B = 1 . Hence the response is
u(t) =
1
1 − ω2
3 cos ωt − (ω2
+ 2) cos t + sin t .
(b)
(a) ω = 0.7 (b) ω = 0.8 (c) ω = 0.9
Note that
u(t) =
6
1 − ω2
sin
(1 − ω)t
2
sin
(ω + 1)t
2
+ cos t + sin t .
15.
(a) ω = 0.7 (b) ω = 0.8 (c) ω = 0.9
3.8 83
18.(a)
(b) Phase plot - u vs u :
84 Chapter 3. Second-Order Linear Equations
Elementary Differential Equations 11th Edition Boyce Solutions Manual
Full Download: https://alibabadownload.com/product/elementary-differential-equations-11th-edition-boyce-solutions-manual/
This sample only, Download all chapters at: AlibabaDownload.com

More Related Content

What's hot

6.4 inverse matrices t
6.4 inverse matrices t6.4 inverse matrices t
6.4 inverse matrices tmath260
 
Contoh bukan subgrup normal
Contoh bukan subgrup normalContoh bukan subgrup normal
Contoh bukan subgrup normalNida Shafiyanti
 
Bai tap nguyen ham tich phan
Bai tap nguyen ham tich phanBai tap nguyen ham tich phan
Bai tap nguyen ham tich phantramhuuduc
 
Capitulo 1, 7ma edición
Capitulo 1, 7ma ediciónCapitulo 1, 7ma edición
Capitulo 1, 7ma ediciónSohar Carr
 
Gamma and betta function harsh shah
Gamma and betta function  harsh shahGamma and betta function  harsh shah
Gamma and betta function harsh shahC.G.P.I.T
 
partial diffrentialequations
partial diffrentialequationspartial diffrentialequations
partial diffrentialequations8laddu8
 
Higher Order Differential Equation
Higher Order Differential EquationHigher Order Differential Equation
Higher Order Differential EquationShrey Patel
 
Tutorial 7 mth 3201
Tutorial 7 mth 3201Tutorial 7 mth 3201
Tutorial 7 mth 3201Drradz Maths
 
Tutorial 3 mth 3201
Tutorial 3 mth 3201Tutorial 3 mth 3201
Tutorial 3 mth 3201Drradz Maths
 
Tutorial 5 mth 3201
Tutorial 5 mth 3201Tutorial 5 mth 3201
Tutorial 5 mth 3201Drradz Maths
 
Nhị thức newton và công thức tổ hợp
Nhị thức newton và công thức tổ hợpNhị thức newton và công thức tổ hợp
Nhị thức newton và công thức tổ hợpThế Giới Tinh Hoa
 
Consistency of linear equations in two and three variables
Consistency of linear equations in two and three variablesConsistency of linear equations in two and three variables
Consistency of linear equations in two and three variablesAamlan Saswat Mishra
 
Persamaan Nonhomogen ; Metode Koefisien Tak ditentukan
Persamaan Nonhomogen ; Metode Koefisien Tak ditentukanPersamaan Nonhomogen ; Metode Koefisien Tak ditentukan
Persamaan Nonhomogen ; Metode Koefisien Tak ditentukanDian Arisona
 
Dethihetmon ptvp thamkhao
Dethihetmon ptvp thamkhaoDethihetmon ptvp thamkhao
Dethihetmon ptvp thamkhaotuongnm
 
MCQs Ordinary Differential Equations
MCQs Ordinary Differential EquationsMCQs Ordinary Differential Equations
MCQs Ordinary Differential EquationsDrDeepaChauhan
 
Engr 213 midterm 1b sol 2009
Engr 213 midterm 1b sol 2009Engr 213 midterm 1b sol 2009
Engr 213 midterm 1b sol 2009akabaka12
 
Roots of Nonlinear Equations - Open Methods
Roots of Nonlinear Equations - Open MethodsRoots of Nonlinear Equations - Open Methods
Roots of Nonlinear Equations - Open MethodsMohammad Tawfik
 
Regula falsi method
Regula falsi methodRegula falsi method
Regula falsi methodandrushow
 

What's hot (20)

Complex numbers 2
Complex numbers 2Complex numbers 2
Complex numbers 2
 
6.4 inverse matrices t
6.4 inverse matrices t6.4 inverse matrices t
6.4 inverse matrices t
 
Contoh bukan subgrup normal
Contoh bukan subgrup normalContoh bukan subgrup normal
Contoh bukan subgrup normal
 
Bai tap nguyen ham tich phan
Bai tap nguyen ham tich phanBai tap nguyen ham tich phan
Bai tap nguyen ham tich phan
 
Capitulo 1, 7ma edición
Capitulo 1, 7ma ediciónCapitulo 1, 7ma edición
Capitulo 1, 7ma edición
 
Gamma and betta function harsh shah
Gamma and betta function  harsh shahGamma and betta function  harsh shah
Gamma and betta function harsh shah
 
partial diffrentialequations
partial diffrentialequationspartial diffrentialequations
partial diffrentialequations
 
Higher Order Differential Equation
Higher Order Differential EquationHigher Order Differential Equation
Higher Order Differential Equation
 
Tutorial 7 mth 3201
Tutorial 7 mth 3201Tutorial 7 mth 3201
Tutorial 7 mth 3201
 
Tutorial 3 mth 3201
Tutorial 3 mth 3201Tutorial 3 mth 3201
Tutorial 3 mth 3201
 
Tutorial 5 mth 3201
Tutorial 5 mth 3201Tutorial 5 mth 3201
Tutorial 5 mth 3201
 
Nhị thức newton và công thức tổ hợp
Nhị thức newton và công thức tổ hợpNhị thức newton và công thức tổ hợp
Nhị thức newton và công thức tổ hợp
 
Consistency of linear equations in two and three variables
Consistency of linear equations in two and three variablesConsistency of linear equations in two and three variables
Consistency of linear equations in two and three variables
 
Persamaan Nonhomogen ; Metode Koefisien Tak ditentukan
Persamaan Nonhomogen ; Metode Koefisien Tak ditentukanPersamaan Nonhomogen ; Metode Koefisien Tak ditentukan
Persamaan Nonhomogen ; Metode Koefisien Tak ditentukan
 
Dethihetmon ptvp thamkhao
Dethihetmon ptvp thamkhaoDethihetmon ptvp thamkhao
Dethihetmon ptvp thamkhao
 
Lec11
Lec11Lec11
Lec11
 
MCQs Ordinary Differential Equations
MCQs Ordinary Differential EquationsMCQs Ordinary Differential Equations
MCQs Ordinary Differential Equations
 
Engr 213 midterm 1b sol 2009
Engr 213 midterm 1b sol 2009Engr 213 midterm 1b sol 2009
Engr 213 midterm 1b sol 2009
 
Roots of Nonlinear Equations - Open Methods
Roots of Nonlinear Equations - Open MethodsRoots of Nonlinear Equations - Open Methods
Roots of Nonlinear Equations - Open Methods
 
Regula falsi method
Regula falsi methodRegula falsi method
Regula falsi method
 

Similar to Elementary Differential Equations 11th Edition Boyce Solutions Manual

Advanced Engineering Mathematics Solutions Manual.pdf
Advanced Engineering Mathematics Solutions Manual.pdfAdvanced Engineering Mathematics Solutions Manual.pdf
Advanced Engineering Mathematics Solutions Manual.pdfWhitney Anderson
 
Antwoorden fourier and laplace transforms, manual solutions
Antwoorden   fourier and laplace transforms, manual solutionsAntwoorden   fourier and laplace transforms, manual solutions
Antwoorden fourier and laplace transforms, manual solutionsHildemar Alvarez
 
Week 8 [compatibility mode]
Week 8 [compatibility mode]Week 8 [compatibility mode]
Week 8 [compatibility mode]Hazrul156
 
Complex roots of the characteristic equation
Complex roots of the characteristic equationComplex roots of the characteristic equation
Complex roots of the characteristic equationTarun Gehlot
 
Lesson 14 a - parametric equations
Lesson 14 a - parametric equationsLesson 14 a - parametric equations
Lesson 14 a - parametric equationsJean Leano
 
Engr 213 midterm 1b sol 2010
Engr 213 midterm 1b sol 2010Engr 213 midterm 1b sol 2010
Engr 213 midterm 1b sol 2010akabaka12
 
Contemporary communication systems 1st edition mesiya solutions manual
Contemporary communication systems 1st edition mesiya solutions manualContemporary communication systems 1st edition mesiya solutions manual
Contemporary communication systems 1st edition mesiya solutions manualto2001
 

Similar to Elementary Differential Equations 11th Edition Boyce Solutions Manual (20)

Advanced Engineering Mathematics Solutions Manual.pdf
Advanced Engineering Mathematics Solutions Manual.pdfAdvanced Engineering Mathematics Solutions Manual.pdf
Advanced Engineering Mathematics Solutions Manual.pdf
 
Sect5 1
Sect5 1Sect5 1
Sect5 1
 
Antwoorden fourier and laplace transforms, manual solutions
Antwoorden   fourier and laplace transforms, manual solutionsAntwoorden   fourier and laplace transforms, manual solutions
Antwoorden fourier and laplace transforms, manual solutions
 
Sect5 2
Sect5 2Sect5 2
Sect5 2
 
Week 8 [compatibility mode]
Week 8 [compatibility mode]Week 8 [compatibility mode]
Week 8 [compatibility mode]
 
Complex roots of the characteristic equation
Complex roots of the characteristic equationComplex roots of the characteristic equation
Complex roots of the characteristic equation
 
Lesson 14 a - parametric equations
Lesson 14 a - parametric equationsLesson 14 a - parametric equations
Lesson 14 a - parametric equations
 
Ch03 4
Ch03 4Ch03 4
Ch03 4
 
Stirling theorem
Stirling theoremStirling theorem
Stirling theorem
 
Ch03 2
Ch03 2Ch03 2
Ch03 2
 
Linear Differential Equations1
Linear Differential Equations1Linear Differential Equations1
Linear Differential Equations1
 
Ch03 3
Ch03 3Ch03 3
Ch03 3
 
Lecture 10
Lecture 10Lecture 10
Lecture 10
 
Lecture 10
Lecture 10Lecture 10
Lecture 10
 
Sect1 1
Sect1 1Sect1 1
Sect1 1
 
Engr 213 midterm 1b sol 2010
Engr 213 midterm 1b sol 2010Engr 213 midterm 1b sol 2010
Engr 213 midterm 1b sol 2010
 
Sect2 1
Sect2 1Sect2 1
Sect2 1
 
Contemporary communication systems 1st edition mesiya solutions manual
Contemporary communication systems 1st edition mesiya solutions manualContemporary communication systems 1st edition mesiya solutions manual
Contemporary communication systems 1st edition mesiya solutions manual
 
Sect1 R
Sect1 RSect1 R
Sect1 R
 
Sect5 4
Sect5 4Sect5 4
Sect5 4
 

Recently uploaded

Proudly South Africa powerpoint Thorisha.pptx
Proudly South Africa powerpoint Thorisha.pptxProudly South Africa powerpoint Thorisha.pptx
Proudly South Africa powerpoint Thorisha.pptxthorishapillay1
 
Blooming Together_ Growing a Community Garden Worksheet.docx
Blooming Together_ Growing a Community Garden Worksheet.docxBlooming Together_ Growing a Community Garden Worksheet.docx
Blooming Together_ Growing a Community Garden Worksheet.docxUnboundStockton
 
Introduction to ArtificiaI Intelligence in Higher Education
Introduction to ArtificiaI Intelligence in Higher EducationIntroduction to ArtificiaI Intelligence in Higher Education
Introduction to ArtificiaI Intelligence in Higher Educationpboyjonauth
 
Framing an Appropriate Research Question 6b9b26d93da94caf993c038d9efcdedb.pdf
Framing an Appropriate Research Question 6b9b26d93da94caf993c038d9efcdedb.pdfFraming an Appropriate Research Question 6b9b26d93da94caf993c038d9efcdedb.pdf
Framing an Appropriate Research Question 6b9b26d93da94caf993c038d9efcdedb.pdfUjwalaBharambe
 
Like-prefer-love -hate+verb+ing & silent letters & citizenship text.pdf
Like-prefer-love -hate+verb+ing & silent letters & citizenship text.pdfLike-prefer-love -hate+verb+ing & silent letters & citizenship text.pdf
Like-prefer-love -hate+verb+ing & silent letters & citizenship text.pdfMr Bounab Samir
 
Organic Name Reactions for the students and aspirants of Chemistry12th.pptx
Organic Name Reactions  for the students and aspirants of Chemistry12th.pptxOrganic Name Reactions  for the students and aspirants of Chemistry12th.pptx
Organic Name Reactions for the students and aspirants of Chemistry12th.pptxVS Mahajan Coaching Centre
 
18-04-UA_REPORT_MEDIALITERAСY_INDEX-DM_23-1-final-eng.pdf
18-04-UA_REPORT_MEDIALITERAСY_INDEX-DM_23-1-final-eng.pdf18-04-UA_REPORT_MEDIALITERAСY_INDEX-DM_23-1-final-eng.pdf
18-04-UA_REPORT_MEDIALITERAСY_INDEX-DM_23-1-final-eng.pdfssuser54595a
 
Presiding Officer Training module 2024 lok sabha elections
Presiding Officer Training module 2024 lok sabha electionsPresiding Officer Training module 2024 lok sabha elections
Presiding Officer Training module 2024 lok sabha electionsanshu789521
 
“Oh GOSH! Reflecting on Hackteria's Collaborative Practices in a Global Do-It...
“Oh GOSH! Reflecting on Hackteria's Collaborative Practices in a Global Do-It...“Oh GOSH! Reflecting on Hackteria's Collaborative Practices in a Global Do-It...
“Oh GOSH! Reflecting on Hackteria's Collaborative Practices in a Global Do-It...Marc Dusseiller Dusjagr
 
ECONOMIC CONTEXT - LONG FORM TV DRAMA - PPT
ECONOMIC CONTEXT - LONG FORM TV DRAMA - PPTECONOMIC CONTEXT - LONG FORM TV DRAMA - PPT
ECONOMIC CONTEXT - LONG FORM TV DRAMA - PPTiammrhaywood
 
Hierarchy of management that covers different levels of management
Hierarchy of management that covers different levels of managementHierarchy of management that covers different levels of management
Hierarchy of management that covers different levels of managementmkooblal
 
MARGINALIZATION (Different learners in Marginalized Group
MARGINALIZATION (Different learners in Marginalized GroupMARGINALIZATION (Different learners in Marginalized Group
MARGINALIZATION (Different learners in Marginalized GroupJonathanParaisoCruz
 
How to Make a Pirate ship Primary Education.pptx
How to Make a Pirate ship Primary Education.pptxHow to Make a Pirate ship Primary Education.pptx
How to Make a Pirate ship Primary Education.pptxmanuelaromero2013
 
Earth Day Presentation wow hello nice great
Earth Day Presentation wow hello nice greatEarth Day Presentation wow hello nice great
Earth Day Presentation wow hello nice greatYousafMalik24
 
Final demo Grade 9 for demo Plan dessert.pptx
Final demo Grade 9 for demo Plan dessert.pptxFinal demo Grade 9 for demo Plan dessert.pptx
Final demo Grade 9 for demo Plan dessert.pptxAvyJaneVismanos
 
CELL CYCLE Division Science 8 quarter IV.pptx
CELL CYCLE Division Science 8 quarter IV.pptxCELL CYCLE Division Science 8 quarter IV.pptx
CELL CYCLE Division Science 8 quarter IV.pptxJiesonDelaCerna
 
POINT- BIOCHEMISTRY SEM 2 ENZYMES UNIT 5.pptx
POINT- BIOCHEMISTRY SEM 2 ENZYMES UNIT 5.pptxPOINT- BIOCHEMISTRY SEM 2 ENZYMES UNIT 5.pptx
POINT- BIOCHEMISTRY SEM 2 ENZYMES UNIT 5.pptxSayali Powar
 
Capitol Tech U Doctoral Presentation - April 2024.pptx
Capitol Tech U Doctoral Presentation - April 2024.pptxCapitol Tech U Doctoral Presentation - April 2024.pptx
Capitol Tech U Doctoral Presentation - April 2024.pptxCapitolTechU
 

Recently uploaded (20)

Proudly South Africa powerpoint Thorisha.pptx
Proudly South Africa powerpoint Thorisha.pptxProudly South Africa powerpoint Thorisha.pptx
Proudly South Africa powerpoint Thorisha.pptx
 
Blooming Together_ Growing a Community Garden Worksheet.docx
Blooming Together_ Growing a Community Garden Worksheet.docxBlooming Together_ Growing a Community Garden Worksheet.docx
Blooming Together_ Growing a Community Garden Worksheet.docx
 
Introduction to ArtificiaI Intelligence in Higher Education
Introduction to ArtificiaI Intelligence in Higher EducationIntroduction to ArtificiaI Intelligence in Higher Education
Introduction to ArtificiaI Intelligence in Higher Education
 
Framing an Appropriate Research Question 6b9b26d93da94caf993c038d9efcdedb.pdf
Framing an Appropriate Research Question 6b9b26d93da94caf993c038d9efcdedb.pdfFraming an Appropriate Research Question 6b9b26d93da94caf993c038d9efcdedb.pdf
Framing an Appropriate Research Question 6b9b26d93da94caf993c038d9efcdedb.pdf
 
OS-operating systems- ch04 (Threads) ...
OS-operating systems- ch04 (Threads) ...OS-operating systems- ch04 (Threads) ...
OS-operating systems- ch04 (Threads) ...
 
Like-prefer-love -hate+verb+ing & silent letters & citizenship text.pdf
Like-prefer-love -hate+verb+ing & silent letters & citizenship text.pdfLike-prefer-love -hate+verb+ing & silent letters & citizenship text.pdf
Like-prefer-love -hate+verb+ing & silent letters & citizenship text.pdf
 
Organic Name Reactions for the students and aspirants of Chemistry12th.pptx
Organic Name Reactions  for the students and aspirants of Chemistry12th.pptxOrganic Name Reactions  for the students and aspirants of Chemistry12th.pptx
Organic Name Reactions for the students and aspirants of Chemistry12th.pptx
 
ESSENTIAL of (CS/IT/IS) class 06 (database)
ESSENTIAL of (CS/IT/IS) class 06 (database)ESSENTIAL of (CS/IT/IS) class 06 (database)
ESSENTIAL of (CS/IT/IS) class 06 (database)
 
18-04-UA_REPORT_MEDIALITERAСY_INDEX-DM_23-1-final-eng.pdf
18-04-UA_REPORT_MEDIALITERAСY_INDEX-DM_23-1-final-eng.pdf18-04-UA_REPORT_MEDIALITERAСY_INDEX-DM_23-1-final-eng.pdf
18-04-UA_REPORT_MEDIALITERAСY_INDEX-DM_23-1-final-eng.pdf
 
Presiding Officer Training module 2024 lok sabha elections
Presiding Officer Training module 2024 lok sabha electionsPresiding Officer Training module 2024 lok sabha elections
Presiding Officer Training module 2024 lok sabha elections
 
“Oh GOSH! Reflecting on Hackteria's Collaborative Practices in a Global Do-It...
“Oh GOSH! Reflecting on Hackteria's Collaborative Practices in a Global Do-It...“Oh GOSH! Reflecting on Hackteria's Collaborative Practices in a Global Do-It...
“Oh GOSH! Reflecting on Hackteria's Collaborative Practices in a Global Do-It...
 
ECONOMIC CONTEXT - LONG FORM TV DRAMA - PPT
ECONOMIC CONTEXT - LONG FORM TV DRAMA - PPTECONOMIC CONTEXT - LONG FORM TV DRAMA - PPT
ECONOMIC CONTEXT - LONG FORM TV DRAMA - PPT
 
Hierarchy of management that covers different levels of management
Hierarchy of management that covers different levels of managementHierarchy of management that covers different levels of management
Hierarchy of management that covers different levels of management
 
MARGINALIZATION (Different learners in Marginalized Group
MARGINALIZATION (Different learners in Marginalized GroupMARGINALIZATION (Different learners in Marginalized Group
MARGINALIZATION (Different learners in Marginalized Group
 
How to Make a Pirate ship Primary Education.pptx
How to Make a Pirate ship Primary Education.pptxHow to Make a Pirate ship Primary Education.pptx
How to Make a Pirate ship Primary Education.pptx
 
Earth Day Presentation wow hello nice great
Earth Day Presentation wow hello nice greatEarth Day Presentation wow hello nice great
Earth Day Presentation wow hello nice great
 
Final demo Grade 9 for demo Plan dessert.pptx
Final demo Grade 9 for demo Plan dessert.pptxFinal demo Grade 9 for demo Plan dessert.pptx
Final demo Grade 9 for demo Plan dessert.pptx
 
CELL CYCLE Division Science 8 quarter IV.pptx
CELL CYCLE Division Science 8 quarter IV.pptxCELL CYCLE Division Science 8 quarter IV.pptx
CELL CYCLE Division Science 8 quarter IV.pptx
 
POINT- BIOCHEMISTRY SEM 2 ENZYMES UNIT 5.pptx
POINT- BIOCHEMISTRY SEM 2 ENZYMES UNIT 5.pptxPOINT- BIOCHEMISTRY SEM 2 ENZYMES UNIT 5.pptx
POINT- BIOCHEMISTRY SEM 2 ENZYMES UNIT 5.pptx
 
Capitol Tech U Doctoral Presentation - April 2024.pptx
Capitol Tech U Doctoral Presentation - April 2024.pptxCapitol Tech U Doctoral Presentation - April 2024.pptx
Capitol Tech U Doctoral Presentation - April 2024.pptx
 

Elementary Differential Equations 11th Edition Boyce Solutions Manual

  • 1. 53 C H A P T E R 3 Second-Order Linear Equations 3.1 1. Let y = ert , so that y = r ert and y = r2 ert . Direct substitution into the differential equation yields (r2 + 2r − 3)ert = 0 . Canceling the exponential, the characteristic equation is r2 + 2r − 3 = 0 . The roots of the equation are r = −3 , 1 . Hence the general solution is y = c1et + c2e−3t . 2. Let y = ert . Substitution of the assumed solution results in the characteristic equation r2 + 3r + 2 = 0 . The roots of the equation are r = −2 , −1 . Hence the general solution is y = c1e−t + c2e−2t . 5. The characteristic equation is 4r2 − 9 = 0 , with roots r = ±3/2 . Therefore the general solution is y = c1e−3t/2 + c2e3t/2 . 6. The characteristic equation is r2 − 2r − 2 = 0 , with roots r = 1 ± √ 3 . Hence the general solution is y = c1e(1− √ 3)t + c2e(1+ √ 3)t . 7. Substitution of the assumed solution y = ert results in the characteristic equa- tion r2 + r − 2 = 0 . The roots of the equation are r = −2 , 1 . Hence the general solution is y = c1e−2t + c2et . Its derivative is y = −2c1e−2t + c2et . Based on the first condition, y(0) = 1 , we require that c1 + c2 = 1 . In order to satisfy y (0) = 1 , we find that −2c1 + c2 = 1 . Solving for the constants, c1 = 0 and c2 = 1 . Hence the specific solution is y(t) = et . It clearly increases without bound as t → ∞. Elementary Differential Equations 11th Edition Boyce Solutions Manual Full Download: https://alibabadownload.com/product/elementary-differential-equations-11th-edition-boyce-solutions-manual/ This sample only, Download all chapters at: AlibabaDownload.com
  • 2. 54 Chapter 3. Second-Order Linear Equations 9. The characteristic equation is r2 + 3r = 0 , with roots r = −3 , 0 . Therefore the general solution is y = c1 + c2e−3t , with derivative y = −3 c2e−3t . In order to satisfy the initial conditions, we find that c1 + c2 = −2 , and −3 c2 = 3 . Hence the specific solution is y(t) = −1 − e−3t . This converges to −1 as t → ∞. 10. The characteristic equation is 2r2 + r − 4 = 0 , with roots r = (−1 ± √ 33 )/4. The general solution is y = c1e(−1− √ 33)t/4 + c2e(−1+ √ 33)t/4 , with derivative y = −1 − √ 33 4 c1e(−1− √ 33)t/4 + −1 + √ 33 4 c2e(−1+ √ 33)t/4 . In order to satisfy the initial conditions, we require that c1 + c2 = 0 and −1 − √ 33 4 c1 + −1 + √ 33 4 c2 = 1. Solving for the coefficients, c1 = −2/ √ 33 and c2 = 2/ √ 33 . The specific solution is y(t) = −2 e(−1− √ 33)t/4 − e(−1+ √ 33)t/4 / √ 33 . It clearly increases without bound as t → ∞.
  • 3. 3.1 55 12. The characteristic equation is 4r2 − 1 = 0 , with roots r = ±1/2 . Therefore the general solution is y = c1e−t/2 + c2et/2 . Since the initial conditions are specified at t = −2 , is more convenient to write y = d1e−(t+2)/2 + d2e(t+2)/2 . The derivative is given by y = − d1e−(t+2)/2 /2 + d2e(t+2)/2 /2 . In order to satisfy the initial conditions, we find that d1 + d2 = 1 , and −d1/2 + d2/2 = −1 . Solving for the coefficients, d1 = 3/2 , and d2 = −1/2 . The specific solution is y(t) = 3 2 e−(t+2)/2 − 1 2 e(t+2)/2 = 3 2e e−t/2 − e 2 et/2 . It clearly decreases without bound as t → ∞. 15. The characteristic equation is 2r2 − 3r + 1 = 0 , with roots r = 1/2 , 1 . There- fore the general solution is y = c1et/2 + c2et , with derivative y = c1et/2 /2 + c2et . In order to satisfy the initial conditions, we require c1 + c2 = 2 and c1/2 + c2 = 1/2 . Solving for the coefficients, c1 = 3 , and c2 = −1 . The specific solution is y(t) = 3et/2 − et . To find the stationary point, set y = 3et/2 /2 − et = 0 . There is a unique solution, with t1 = ln(9/4). The maximum value is then y(t1) = 9/4. To find the x-intercept, solve the equation 3et/2 − et = 0 . The solution is readily found to be t2 = ln 9 ≈ 2.1972 . 17. The characteristic equation is r2 − (2α − 1)r + α(α − 1) = 0 . Examining the coefficients, the roots are r = α , α − 1 . Hence the general solution of the differen- tial equation is y(t) = c1eαt + c2e(α−1)t . Assuming α ∈ R , all solutions will tend to zero as long as α < 0 . On the other hand, all solutions will become unbounded as long as α − 1 > 0 , that is, α > 1 .
  • 4. 56 Chapter 3. Second-Order Linear Equations 19.(a) The characteristic roots are r = −3 , −2 . The solution of the initial value problem is y(t) = (6 + β)e−2t − (4 + β)e−3t . (b) The maximum point has coordinates t0 = ln [(3(4 + β))/(2(6 + β))], y0 = 4(6 + β)3 /(27(4 + β)2 ) . (c) y0 = 4(6 + β)3 /(27(4 + β)2 ) ≥ 4 , as long as β ≥ 6 + 6 √ 3 . (d) limβ→∞ t0 = ln(3/2), limβ→∞ y0 = ∞ . 20.(a) Assuming that y is a constant, the differential equation reduces to c y = d . Hence the only equilibrium solution is y = d/c . (b) Setting y = Y + d/c , substitution into the differential equation results in the equation a Y + b Y + c(Y + d/c ) = d. The equation satisfied by Y is a Y + b Y + cY = 0. 3.2 1. W(e2t , e−3t/2 ) = e2t e−3t/2 2e2t −3 2 e−3t/2 = − 7 2 et/2 . 3. W(e−2t , t e−2t ) = e−2t te−2t −2e−2t (1 − 2t)e−2t = e−4t . 4. W(et sin t , et cos t) = et sin t et cos t et (sin t + cos t) et (cos t − sin t) = −e2t . 5. W(cos2 θ , 1 + cos 2θ) = cos2 θ 1 + cos 2θ −2 sin θ cos θ −2 sin 2θ = 0 . 6. Write the equation as y + (3/t)y = 1 . p(t) = 3/t is continuous for all t > 0 . Since t0 > 0 , the IVP has a unique solution for all t > 0 . 7. Write the equation as y + (3/(t − 4))y + (4/t(t − 4))y = 2/t(t − 4) . The coef- ficients are not continuous at t = 0 and t = 4 . Since t0 ∈ (0 , 4) , the largest interval is 0 < t < 4 . 8. The coefficient 3 ln |t| is discontinuous at t = 0 . Since t0 > 0 , the largest interval of existence is 0 < t < ∞ .
  • 5. 3.2 57 10. y1 = 2 . We see that t2 (2) − 2(t2 ) = 0 . y2 = 2 t−3 , with t2 (y2 ) − 2(y2) = 0 . Let y3 = c1t2 + c2t−1 , then y3 = 2c1 + 2c2t−3 . It is evident that y3 is also a solution. 13. No. Substituting y = sin(t2 ) into the differential equation, −4t2 sin(t2 ) + 2 cos(t2 ) + 2t cos(t2 )p(t) + sin(t2 )q(t) = 0 . At t = 0, this equation becomes 2 = 0 (if we suppose that p(t) and q(t) are contin- uous), which is impossible. 14. W(e2t , g(t)) = e2t g (t) − 2e2t g(t) = 3e4t . Dividing both sides by e2t , we find that g must satisfy the ODE g − 2g = 3e2t . Hence g(t) = 3t e2t + c e2t . 15. W(f , g) = fg − f g = t cos t − sin t , and W(u , v) = −4fg + 4f g . Hence W(u , v) = −4t cos t + 4 sin t . 16. We compute W(a1y1 + a2y2, b1y1 + b2y2) = a1y1 + a2y2 b1y1 + b2y2 a1y1 + a2y2 b1y1 + b2y2 = = (a1y1 + a2y2)(b1y1 + b2y2) − (b1y1 + b2y2)(a1y1 + a2y2) = = a1b2(y1y2 − y1y2) − a2b1(y1y2 − y1y2) = (a1b2 − a2b1)W(y1, y2). This now readily shows that y3 and y4 form a fundamental set of solutions if and only if a1b2 − a2b1 = 0. 18. The general solution is y = c1e−3t + c2e−t . W(e−3t , e−t ) = 2e−4t , and hence the exponentials form a fundamental set of solutions. On the other hand, the fun- damental solutions must also satisfy the conditions y1(1) = 1, y1(1) = 0; y2(1) = 0, y2(1) = 1. For y1, the initial conditions require c1 + c2 = e, −3c1 − c2 = 0. The coefficients are c1 = −e3 /2, c2 = 3e/2. For the solution y2, the initial conditions re- quire c1 + c2 = 0, −3c1 − c2 = e. The coefficients are c1 = −e3 /2, c2 = e/2. Hence the fundamental solutions are y1 = − 1 2 e−3(t−1) + 3 2 e−(t−1) and y2 = − 1 2 e−3(t−1) + 1 2 e−(t−1) . 19. Yes. y1 = −4 cos 2t ; y2 = −4 sin 2t . W(cos 2t , sin 2t) = 2 . 20. Clearly, y1 = et is a solution. y2 = (1 + t)et , y2 = (2 + t)et . Substitution into the ODE results in (2 + t)et − 2(1 + t)et + t et = 0 . Furthermore, W(et , tet ) = e2t . Hence the solutions form a fundamental set of solutions. 24. Writing the equation in standard form, we find that P(t) = sin t/ cos t . Hence the Wronskian is W(t) = c e− (sin t/ cos t) dt = c eln|cos t| = c cos t , in which c is some constant.
  • 6. 58 Chapter 3. Second-Order Linear Equations 25. Writing the equation in standard form, we find that P(x) = −2x/(1 − x2 ). The Wronskian is W(x) = c e− −2x/(1−x2 ) dx = c e− ln|1−x2 | = c/(1 − x2 ), in which c is some constant. 26. Rewrite the equation as p(t)y + p (t)y + q(t)y = 0 . After writing the equa- tion in standard form, we have P(t) = p (t)/p(t) . Hence the Wronskian is W(t) = c e− p (t)/p(t) dt = c e− ln p(t) = c/p(t) . 28. For the given differential equation, the Wronskian satisfies the first order dif- ferential equation W + p(t)W = 0 . Given that W is constant, it is necessary that p(t) ≡ 0 . 32. P = 1 , Q = x , R = 1 . We have P − Q + R = 0 . The equation is exact. Note that (y ) + (xy) = 0 . Hence y + xy = c1 . This equation is linear, with integrating factor µ = ex2 /2 . Therefore the general solution is y(x) = c1e−x2 /2 x x0 eu2 /2 du + c2e−x2 /2 . 34. P = x2 , Q = x, R = −1. We have P − Q + R = 0. The equation is exact. Write the equation as (x2 y ) − (xy) = 0. After integration, we conclude that x2 y − xy = c. Divide both sides of the differential equation by x2 . The resulting equation is linear, with integrating factor µ = 1/x . Hence (y/x) = c x−3 . The solution is y(t) = c1x−1 + c2x . 36. P = x2 , Q = x , R = x2 − ν2 . Hence the coefficients are 2P − Q = 3x and P − Q + R = x2 + 1 − ν2 . The adjoint of the original differential equation is given by x2 µ + 3x µ + (x2 + 1 − ν2 )µ = 0 . 37. P = 1 , Q = 0 , R = −x . Hence the coefficients are given by 2P − Q = 0 and P − Q + R = −x . Therefore the adjoint of the original equation is µ − x µ = 0 . 3.3 1. e2−3i = e2 e−3i = e2 (cos 3 − i sin 3). 2. eiπ = cos π + i sin π = −1 . 3. e2−(π/2) i = e2 (cos(π/2) − i sin(π/2)) = −e2 i . 6. The characteristic equation is r2 − 2r + 6 = 0 , with roots r = 1 ± i √ 5 . Hence the general solution is y = c1et cos √ 5 t + c2 et sin √ 5 t. 7. The characteristic equation is r2 + 2r + 2 = 0 , with roots r = −1 ± i. Hence the general solution is y = c1e−t cos t + c2 e−t sin t.
  • 7. 3.3 59 9. The characteristic equation is r2 + 2r + 1.25 = 0, with roots r = −1 ± i/2. Hence the general solution is y = c1e−t cos(t/2) + c2 e−t sin(t/2). 11. The characteristic equation is r2 + 4r + 6.25 = 0 , with roots r = −2 ± (3/2) i. Hence the general solution is y = c1e−2t cos(3t/2) + c2 e−2t sin(3t/2). 12. The characteristic equation is r2 + 4 = 0 , with roots r = ± 2i. Hence the general solution is y = c1 cos 2t + c2 sin 2t . Now y = −2c1 sin 2t + 2c2 cos 2t . Based on the first condition, y(0) = 0 , we require that c1 = 0 . In order to satisfy the condition y (0) = 1 , we find that 2c2 = 1 . The constants are c1 = 0 and c2 = 1/2 . Hence the specific solution is y(t) = sin 2t /2. The solution is periodic. 13. The characteristic equation is r2 − 2r + 5 = 0 , with roots r = 1 ± 2i. Hence the general solution is y = c1et cos 2t + c2 et sin 2t . Based on the initial condition y(π/2) = 0 , we require that c1 = 0 . It follows that y = c2 et sin 2t , and so the first derivative is y = c2 et sin 2t + 2c2 et cos 2t . In order to satisfy the condition y (π/2) = 2, we find that −2eπ/2 c2 = 2 . Hence we have c2 = −e−π/2 . There- fore the specific solution is y(t) = −et−π/2 sin 2t . The solution oscillates with an exponentially growing amplitude. 14. The characteristic equation is r2 + 1 = 0 , with roots r = ± i. Hence the gen- eral solution is y = c1 cos t + c2 sin t . Its derivative is y = −c1 sin t + c2 cos t . Based on the first condition, y(π/3) = 2 , we require that c1 + √ 3 c2 = 4 . In or- der to satisfy the condition y (π/3) = −4 , we find that − √ 3 c1 + c2 = −8 . Solving
  • 8. 60 Chapter 3. Second-Order Linear Equations these for the constants, c1 = 1 + 2 √ 3 and c2 = √ 3 − 2 . Hence the specific solution is a steady oscillation, given by y(t) = (1 + 2 √ 3) cos t + ( √ 3 − 2) sin t . 17.(a) The characteristic equation is 5r2 + 2r + 7 = 0 , with roots r = −(1 ± i √ 34)/5. The solution is u = c1e−t/5 cos √ 34t/5 + c2e−t/5 sin √ 34t/5 . Invoking the given initial conditions, we obtain the equations for the coefficients : c1 = 2 , −2 + √ 34 c2 = 5 . That is, c1 = 2 , c2 = 7/ √ 34 . Hence the specific solution is u(t) = 2e−t/5 cos √ 34 5 t + 7 √ 34 e−t/5 sin √ 34 5 t . (b) Based on the graph of u(t), T is in the interval 14 < t < 16 . A numerical solution on that interval yields T ≈ 14.5115 . 19. Direct calculation gives the result. On the other hand, it can be shown that W(f g , f h) = f2 W(g , h). Hence W(eλt cos µt , eλt sin µt) = e2λt W(cos µt , sin µt) = e2λt [cos µt(sin µt) − (cos µt) sin µt] = µ e2λt . 20.(a) Clearly, y1 and y2 are solutions. Also, W(cos t, sin t) = cos2 t + sin2 t = 1. (b) y = i eit , y = i2 eit = −eit . Evidently, y is a solution and so y = c1y1 + c2y2. (c) Setting t = 0 , 1 = c1 cos 0 + c2 sin 0 , and c1 = 1.
  • 9. 3.3 61 (d) Differentiating, i eit = c2 cos t . Setting t = 0 , i = c2 cos 0 and hence c2 = i . Therefore eit = cos t + i sin t . 21. Euler’s formula is eit = cos t + i sin t . It follows that e−it = cos t − i sin t . Adding these equation, eit + e−it = 2 cos t . Subtracting the two equations results in eit − e−it = 2i sin t . 22. Let r1 = λ1 + iµ1 , and r2 = λ2 + iµ2 . Then e(r1+r2)t = e(λ1+λ2)t+i(µ1+µ2)t = e(λ1+λ2)t [cos(µ1 + µ2)t + i sin(µ1 + µ2)t] = = e(λ1+λ2)t [(cos µ1t + i sin µ1t)(cos µ2t + i sin µ2t)] = = eλ1t (cos µ1t + i sin µ1t) · eλ2t (cos µ1t + i sin µ1t) = er1t er2t . Hence e(r1+r2)t = er1t er2t . 23. Clearly, u = λeλt cos µt − µeλt sin µt = eλt (λ cos µt − µ sin µt) and then u = λeλt (λ cos µt − µ sin µt) + eλt (−λµ sin µt − µ2 cos µt). Plugging these into the dif- ferential equation, dividing by eλt = 0 and arranging the sine and cosine terms we obtain that the identity to prove is (a(λ2 − µ2 ) + bλ + c) cos µt + (−2λµa − bµ) sin µt = 0. We know that λ ± iµ solves the characteristic equation ar2 + br + c = 0, so a(λ − iµ)2 + b(λ − iµ) + c = a(λ2 − µ2 ) + bλ + c + i(−2λµa − µb) = 0. If this complex number is zero, then both the real and imaginary parts of it are zero, but those are the coefficients of cos µt and sin µt in the above identity, which proves that au + bu + cu = 0. The solution for v is analogous. 26. The equation transforms into y + y = 0. The characteristic roots are r = ±i. The solution is y = c1 cos(x) + c2 sin(x) = c1 cos(ln t) + c2 sin(ln t). 28. The equation transforms into y − 5y − 6y = 0. The characteristic roots are r = −1, 6. The solution is y = c1e−x + c2e6x = c1e− ln t + c2e6 ln t = c1/t + c2t6 . 29. The equation transforms into y − 5y + 6y = 0. The characteristic roots are r = 2, 3. The solution is y = c1e2x + c2e3x = c1e2 ln t + c2e3 ln t = c1t2 + c2t3 . 30. The equation transforms into y + 2y − 3y = 0. The characteristic roots are r = 1, −3. The solution is y = c1ex + c2e−3x = c1eln t + c2e−3 ln t = c1t + c2/t3 . 31. The equation transforms into y + 6y + 10y = 0. The characteristic roots are r = −3 ± i. The solution is y = c1e−3x cos(x) + c2e−3x sin(x) = c1 1 t3 cos(ln t) + c2 1 t3 sin(ln t). 32.(a) By the chain rule, y (x) = (dy/dx)x . In general, dz/dt = (dz/dx)(dx/dt). Setting z = (dy/dt), we have d2 y dt2 = dz dx dx dt = d dx dy dx dx dt dx dt = d2 y dx2 dx dt dx dt + dy dx d dx dx dt dx dt .
  • 10. 62 Chapter 3. Second-Order Linear Equations However, d dx dx dt dx dt = d2 x dt2 dt dx · dx dt = d2 x dt2 . Hence d2 y dt2 = d2 y dx2 dx dt 2 + dy dx d2 x dt2 . (b) Substituting the results in part (a) into the general differential equation, y + p(t)y + q(t)y = 0 , we find that d2 y dx2 dx dt 2 + dy dx d2 x dt2 + p(t) dy dx dx dt + q(t)y = 0 . Collecting the terms, dx dt 2 d2 y dx2 + d2 x dt2 + p(t) dx dt dy dx + q(t)y = 0 . (c) Assuming (dx/dt)2 = k q(t) , and q(t) > 0 , we find that dx/dt = k q(t) , which can be integrated. That is, x = u(t) = k q(t) dt = q(t) dt, since k = 1. (d) Let k = 1 . It follows that d2 x/dt2 + p(t)dx/dt = du/dt + p(t)u(t) = q /2 √ q + p √ q . Hence d2 x dt2 + p(t) dx dt / dx dt 2 = q (t) + 2p(t)q(t) 2 [q(t)] 3/2 . As long as dx/dt = 0 , the differential equation can be expressed as d2 y dx2 + q (t) + 2p(t)q(t) 2 [q(t)] 3/2 dy dx + y = 0. (e) To find the analogue to the condition found in part d) for the case when q(t) < 0 we return to the conditions that make the coefficients on y, dy/dt and d2 y/dt2 proportional to each other. Since the coefficients on y and d2 y/dt2 are propor- tional, (dx/dt)2 = αq(t), and we may take α = −1. Thus dx/dt = (−q(t))1/2 and d2 y/dt2 = (−q /2)(−q)−1/2 . Since the coefficients on y and dy/dt are proportional, there is a constant β with β q = d2 y dt2 + p(t) dx dt = −q 2 (−q)−1/2 + p(−q)1/2 = −q − 2pq 2(−q)1/2 and dividing each side of the equation by −q gives −β = −q − 2pq 2(−q)3/2 , or 2β = q + 2pq (−q)3/2 Thus the desired condition is that (q + 2pq)/(−q)3/2 must be a constant.
  • 11. 3.4 63 34. Note that p(t) = 3t and q(t) = t2 . We have x = t dt = t2 /2 . Furthermore, q (t) + 2p(t)q(t) 2 [q(t)] 3/2 = 1 + 3t2 t2 . The ratio is not constant, and therefore the equation cannot be transformed. 35. Note that p(t) = t − 1/t and q(t) = t2 . We have x = t dt = t2 /2 . Further- more, q (t) + 2p(t)q(t) 2 [q(t)] 3/2 = 1 . The ratio is constant, and therefore the equation can be transformed. From Problem 32, the transformed equation is d2 y dx2 + dy dx + y = 0 . Based on the methods in this section, the characteristic equation is r2 + r + 1 = 0 , with roots r = (−1 ± i √ 3)/2. The general solution is y(x) = c1e−x/2 cos √ 3 x/2 + c2 e−x/2 sin √ 3 x/2. Since x = t2 /2 , the solution in the original variable t is y(t) = e−t2 /4 c1 cos ( √ 3 t2 /4) + c2 sin ( √ 3 t2 /4) . 36. Note that p(t) = t and q(t) = −e−t2 < 0 for −∞ < t < ∞. To proceed we must confirm that (q + 2pq)/(−q)3/2 is a constant: q + 2pq (−q)3/2 = 2te−t2 + 2t(−et2 ) (e−t2 )3/2 = 0. Thus the differential equation can be transformed into an equation with constant coefficients by letting x = u(t) = e−t2 /2 dt. Substituting x = u(t) in the differ- ential equation found in part (b) of Problem 32 we obtain, after dividing by the coefficient of d2 y/dx2 , the differential equation (d2 y/dx2 ) − y = 0. Hence the gen- eral solution of the original differential equation is y(t) = c1ex(t) + c2e−x(t) , where x(t) = e−t2 /2 dt. 3.4 2. The characteristic equation is 9r2 + 6r + 1 = 0 , with the double root r = −1/3 . The general solution is y(t) = c1e−t/3 + c2t e−t/3 . 3. The characteristic equation is 4r2 − 4r − 3 = 0 , with roots r = −1/2 , 3/2 . The general solution is y(t) = c1e−t/2 + c2e3t/2 . 5. The characteristic equation is r2 − 6r + 9 = 0 , with the double root r = 3 . The general solution is y(t) = c1e3t + c2t e3t .
  • 12. 64 Chapter 3. Second-Order Linear Equations 6. The characteristic equation is 4r2 + 17r + 4 = 0 , with roots r = −1/4 , −4 . The general solution is y(t) = c1e−t/4 + c2e−4t . 7. The characteristic equation is 16r2 + 24r + 9 = 0, with double root r = −3/4. The general solution is y(t) = c1e−3t/4 + c2t e−3t/4 . 8. The characteristic equation is 2r2 + 2r + 1 = 0. We obtain the complex roots r = (−1 ± i)/2. The general solution is y(t) = c1e−t/2 cos(t/2) + c2e−t/2 sin(t/2). 9. The characteristic equation is 9r2 − 12r + 4 = 0 , with the double root r = 2/3 . The general solution is y(t) = c1e2t/3 + c2t e2t/3 . Invoking the first initial condi- tion, it follows that c1 = 2 . Now y (t) = (4/3 + c2)e2t/3 + 2c2t e2t/3 /3 . Invoking the second initial condition, 4/3 + c2 = −1 , or c2 = −7/3 . Hence we obtain the solution y(t) = 2e2t/3 − (7/3)te2t/3 . Since the second term dominates for large t, y(t) → −∞. 12. The characteristic roots are r1 = r2 = 1/2 . Hence the general solution is given by y(t) = c1et/2 + c2t et/2 . Invoking the initial conditions, we require that c1 = 2 , and that 1 + c2 = b. The specific solution is y(t) = 2et/2 + (b − 1)t et/2 . Since the second term dominates, the long-term solution depends on the sign of the coefficient b − 1. The critical value is b = 1. 15.(a) The characteristic equation is r2 + 2a r + a2 = (r + a)2 = 0 . (b) With p(t) = 2a , Abel’s Formula becomes W(y1 , y2) = c e− 2a dt = c e−2at . (c) y1(t) = e−at is a solution. From part (b), with c = 1, e−at y2(t) + a e−at y2(t) = e−2at , which can be written as (eat y2(t)) = 1, resulting in eat y2(t) = t. 17.(a) If the characteristic equation ar2 + br + c has equal roots r1, then ar2 1 + br1 + c = a(r − r1)2 = 0. Then clearly L[ert ] = (ar2 + br + c)ert = a(r − r1)2 ert . This gives immediately that L[er1t ] = 0. (b) Differentiating the identity in part (a) with respect to r we get (2ar + b)ert + (ar2 + br + c)tert = 2a(r − r1)ert + a(r − r1)2 tert . Again, this gives L[ter1t ] = 0.
  • 13. 3.4 65 18. Set y2(t) = t2 v(t) . Substitution into the differential equation results in t2 (t2 v + 4tv + 2v) − 4t(t2 v + 2tv) + 6t2 v = 0 . After collecting terms, we end up with t4 v = 0 . Hence v(t) = c1 + c2t , and thus y2(t) = c1t2 + c2t3 . Setting c1 = 0 and c2 = 1 , we obtain y2(t) = t3 . 19. Set y2(t) = t v(t) . Substitution into the differential equation results in t2 (tv + 2v ) + 2t(tv + v) − 2tv = 0 . After collecting terms, we end up with t3 v + 4t2 v = 0 . This equation is linear in the variable w = v . It follows that v (t) = c t−4 , and v(t) = c1t−3 + c2 . Thus y2(t) = c1t−2 + c2t . Setting c1 = 1 and c2 = 0 , we obtain y2(t) = t−2 . 23. Direct substitution verifies that y1(t) = e−δx2 /2 is a solution of the differential equation. Now set y2(x) = y1(x) v(x). Substitution of y2 into the equation results in v − δxv = 0. This equation is linear in the variable w = v . An integrating factor is µ = e−δx2 /2 . Rewrite the equation as [ e−δx2 /2 v ] = 0 , from which it follows that v (x) = c1 eδx2 /2 . Integrating, we obtain v(x) = c1 x 0 eδu2 /2 du + v(0). Hence y2(x) = c1e−δx2 /2 x 0 eδu2 /2 du + c2e−δx2 /2 . Setting c2 = 0 , we obtain a second independent solution. 25. After writing the differential equation in standard form, we have p(t) = 3/t . Based on Abel’s identity, W(y1, y2) = c1e− 3/t dt = c1t−3 . As shown in Problem 24, two solutions of a second order linear equation satisfy (y2/y1) = W(y1, y2)/y2 1. In the given problem, y1(t) = t−1 . Hence (t y2) = c1t−1 . Integrating both sides of the equation, y2(t) = c1t−1 ln t + c2t−1 . Setting c1 = 1 and c2 = 0 we obtain y2(t) = t−1 ln t. 27. Write the differential equation in standard form to find p(x) = 1/x. Based on Abel’s identity, W(y1, y2) = c e− 1/x dx = c x−1 . Two solutions of a second order linear differential equation satisfy (y2/y1) = W(y1, y2)/y2 1 . In the given problem, y1(x) = x−1/2 sin x . Hence ( √ x sin x y2) = c 1 sin2 x . Integrating both sides of the equation, y2(x) = c1x−1/2 cos x + c2x−1/2 sin x. Set- ting c1 = 1 and c2 = 0 , we obtain y2(x) = x−1/2 cos x . 29.(a) The characteristic equation is ar2 + c = 0 . If a , c > 0 , then the roots are r = ± i c/a . The general solution is y(t) = c1 cos c a t + c2 sin c a t ,
  • 14. 66 Chapter 3. Second-Order Linear Equations which is bounded. (b) The characteristic equation is ar2 + br = 0 . The roots are r = 0 , −b/a , and hence the general solution is y(t) = c1 + c2e−bt/a . Clearly, y(t) → c1 . With the given initial conditions, c1 = y0 + (a/b)y0. 30. Note that 2 cos t sin t = sin 2t. Then 1 − k cos t sin t = 1 − (k/2) sin 2t . Now if 0 < k < 2 , then (k/2) sin 2t < |sin 2t| and −(k/2) sin 2t > − |sin 2t|. Hence 1 − k cos t sin t = 1 − k 2 sin 2t > 1 − |sin 2t| ≥ 0 . 31. The equation transforms into y − 4y + 4y = 0. We obtain a double root r = 2. The solution is y = c1e2x + c2xe2x = c1e2 ln t + c2 ln te2 ln t = c1t2 + c2t2 ln t. 33. The equation transforms into y + 2y + y = 0. We get a double root r = −1. The solution is y = c1e−x + c2xe−x = c1e− ln t + c2 ln te− ln t = c1t−1 + c2t−1 ln t. 34. The equation transforms into y − 3y + 9y/4 = 0. We obtain the double root r = 3/2. The solution is y = c1e3x/2 + c2xe3x/2 = c1e3 ln t/2 + c2 ln te3 ln t/2 = c1t3/2 + c2t3/2 ln t. 3.5 2. The characteristic equation for the homogeneous problem is r2 − r − 2 = 0 , with roots r = −1, 2. Hence yc(t) = c1e−t + c2e2t . Set Y = At2 + Bt + C. Substitution into the given differential equation, and comparing the coefficients, results in the system of equations −2A = 4, −2A − 2B = −2 and 2A − B − 2C = 0. Hence Y = −2t2 + 3t − 7/2. The general solution is y(t) = yc(t) + Y . 3. The characteristic equation for the homogeneous problem is r2 + r − 6 = 0 , with roots r = −3, 2. Hence yc(t) = c1e−3t + c2e2t . Set Y = Ae3t + Be−2t . Substitu- tion into the given differential equation, and comparing the coefficients, results in the system of equations 6A = 12 and −4B = 12. Hence Y = 2e3t − 3e−2t . The general solution is y(t) = yc(t) + Y . 4. The characteristic equation for the homogeneous problem is r2 − 2r − 3 = 0 , with roots r = −1 , 3 . Hence yc(t) = c1e−t + c2e3t . Note that the assignment Y = Ate−t is not sufficient to match the coefficients. Try Y = Ate−t + Bt2 e−t . Substitution into the differential equation, and comparing the coefficients, results in the system of equations −4A + 2B = 0 and −8B = −3 . This implies that Y = (3/16)te−t + (3/8)t2 e−t . The general solution is y(t) = yc(t) + Y . 8. The characteristic equation for the homogeneous problem is r2 + ω2 0 = 0 , with complex roots r = ± ω0 i. Hence yc(t) = c1 cos ω0 t + c2 sin ω0 t . Since ω = ω0 , set Y = A cos ωt + B sin ωt . Substitution into the ODE and comparing the co- efficients results in the system of equations (ω2 0 − ω2 )A = 1 and (ω2 0 − ω2 )B = 0 .
  • 15. 3.5 67 Hence Y = 1 ω2 0 − ω2 cos ωt . The general solution is y(t) = yc(t) + Y . 9. From Problem 8, yc(t) is known. Since cos ω0 t is a solution of the homogeneous problem, set Y = At cos ω0 t + Bt sin ω0 t . Substitution into the given ODE and comparing the coefficients results in A = 0 and B = 1/2ω0 . Hence the general solution is y(t) = c1 cos ω0 t + c2 sin ω0 t + t sin ω0 t/(2ω0 ). 12. The characteristic equation for the homogeneous problem is r2 + 4 = 0 , with roots r = ± 2i. Hence yc(t) = c1 cos 2t + c2 sin 2t . Set Y1 = A + Bt + Ct2 . Com- paring the coefficients of the respective terms, we find that A = −1/8 , B = 0 , C = 1/4 . Now set Y2 = D et , and obtain D = 3/5 . Hence the general solution is y(t) = c1 cos 2t + c2 sin 2t − 1/8 + t2 /4 + 3 et /5. Invoking the initial conditions, we require that 19/40 + c1 = 0 and 3/5 + 2c2 = 2 . Hence c1 = −19/40 and c2 = 7/10 . 13. The characteristic equation for the homogeneous problem is r2 − 2r + 1 = 0 , with a double root r = 1 . Hence yc(t) = c1et + c2t et . Consider g1(t) = t et . Note that g1 is a solution of the homogeneous problem. Set Y1 = At2 et + Bt3 et (the first term is not sufficient for a match). Upon substitution, we obtain Y1 = t3 et /6 . By inspection, Y2 = 4 . Hence the general solution is y(t) = c1et + c2t et + t3 et /6 + 4 . Invoking the initial conditions, we require that c1 + 4 = 1 and c1 + c2 = 1 . Hence c1 = −3 and c2 = 4 . 14. The characteristic equation for the homogeneous problem is r2 + 4 = 0 , with roots r = ±2i. Hence yc(t) = c1 cos 2t + c2 sin 2t . Since the function sin 2t is a solution of the homogeneous problem, set Y = At cos 2t + Bt sin 2t . Upon substitution, we obtain Y = −3t cos 2t /4. Hence the general solution is y(t) = c1 cos 2t + c2 sin 2t − 3t cos 2t /4. Invoking the initial conditions, we require that c1 = 2 and 2c2 − (3/4) = −1 . Hence c1 = 2 and c2 = −1/8 . 15. The characteristic equation for the homogeneous problem is r2 + 2r + 5 = 0 , with complex roots r = −1 ± 2i. Hence yc(t) = c1e−t cos 2t + c2e−t sin 2t . Based on the form of g(t), set Y = At e−t cos 2t + Bt e−t sin 2t . After compar- ing coefficients, we obtain Y = t e−t sin 2t . Hence the general solution is y(t) = c1e−t cos 2t + c2e−t sin 2t + t e−t sin 2t. Invoking the initial conditions, we require that c1 = 1 and −c1 + 2c2 = 0 . Hence c1 = 1 and c2 = 1/2 . 17.(a) The characteristic equation for the homogeneous problem is r2 − 5r + 6 = 0 , with roots r = 2 , 3. Hence yc(t) = c1e2t + c2e3t . Consider g1(t) = e2t (3t + 4) sin t , and g2(t) = et cos 2t. Based on the form of these functions on the right hand side of the ODE, set Y2(t) = et (A1 cos 2t + A2 sin 2t) and Y1(t) = (B1 + B2t)e2t sin t + (C1 + C2t)e2t cos t. (b) Substitution into the equation and comparing the coefficients results in Y (t) = − 1 20 (et cos 2t + 3et sin 2t) + 3 2 te2t (cos t − sin t) + e2t ( 1 2 cos t − 5 sin t).
  • 16. 68 Chapter 3. Second-Order Linear Equations 19.(a) The homogeneous solution is yc(t) = c1 cos 2t + c2 sin 2t. Since cos 2t and sin 2t are both solutions of the homogeneous equation, set Y (t) = t(A0 + A1t + A2t2 ) cos 2t + t(B0 + B1t + B2t2 ) sin 2t . (b) Substitution into the equation and comparing the coefficients results in Y (t) = ( 13 32 t − 1 12 t3 ) cos 2t + 1 16 (28t + 13t2 ) sin 2t . 20.(a) The homogeneous solution is yc(t) = c1e−t + c2te−2t . None of the functions on the right hand side are solutions of the homogenous equation. In order to include all possible combinations of the derivatives, consider Y (t) = et (A0 + A1t + A2t2 ) cos 2t + et (B0 + B1t + B2t2 ) sin 2t + + e−t (C1 cos t + C2 sin t) + Det . (b) Substitution into the differential equation and comparing the coefficients results in Y (t) = et (A0 + A1t + A2t2 ) cos 2t + et (B0 + B1t + B2t2 ) sin 2t + e−t (− 3 2 cos t + 3 2 sin t) + 2et /3, in which A0 = −4105/35152, A1 = 73/676, A2 = −5/52, B0 = −1233/35152, B1 = 10/169, B2 = 1/52. 21.(a) The homogeneous solution is yc(t) = c1e−t cos 2t + c2e−t sin 2t. None of the terms on the right hand side are solutions of the homogenous equation. In order to include the appropriate combinations of derivatives, consider Y (t) = e−t (A1t + A2t2 ) cos 2t + e−t (B1t + B2t2 ) sin 2t + + e−2t (C0 + C1t) cos 2t + e−2t (D0 + D1t) sin 2t. (b) Substitution into the differential equation and comparing the coefficients results in Y (t) = 3 16 te−t cos 2t + 3 8 t2 e−t sin 2t − 1 25 e−2t (7 + 10t) cos 2t + 1 25 e−2t (1 + 5t) sin 2t . 23. The homogeneous solution is yc(t) = c1 cos λt + c2 sin λt. Since the differential operator does not contain a first derivative (and λ = mπ), we can set Y (t) = N m=1 Cm sin mπt .
  • 17. 3.5 69 Substitution into the differential equation yields − N m=1 m2 π2 Cm sin mπt + λ2 N m=1 Cm sin mπt = N m=1 am sin mπt . Equating coefficients of the individual terms, we obtain Cm = am λ2 − m2π2 , m = 1, 2 . . . N. 25. Since a , b, c > 0 , the roots of the characteristic equation have negative real parts. That is, r = α ± β i , where α < 0 . Hence the homogeneous solution is yc(t) = c1eαt cos βt + c2eαt sin βt . If g(t) = d , then the general solution is y(t) = d/c + c1eαt cos βt + c2eαt sin βt . Since α < 0 , y(t) → d/c as t → ∞ . If c = 0, then the characteristic roots are r = 0 and r = −b/a . The ODE becomes ay + by = d . Integrating both sides, we find that ay + by = d t + c1. The general solution can be expressed as y(t) = d t/b + c1 + c2e−bt/a . In this case, the solution grows without bound. If b = 0, also, then the differential equation can be written as y = d/a , which has general solution y(t) = d t2 /2a + c1 + c2 . Hence the assertion is true only if the coefficients are positive. 27.(a) Since D is a linear operator, D2 y + bDy + cy = D2 y − (r1 + r2)Dy + r1r2y = D2 y − r2Dy − r1Dy + r1r2y = D(Dy − r2y) − r1(Dy − r2y) = (D − r1)(D − r2)y. (b) Let u = (D − r2)y . Then the ODE (i) can be written as (D − r1)u = g(t), that is, u − r1u = g(t). The latter is a linear first order equation in u . Its general solution is u(t) = er1t t t0 e−r1τ g(τ)dτ + c1er1t . From above, we have y − r2y = u(t). This equation is also a first order ODE. Hence the general solution of the original second order equation is y(t) = er2t t t0 e−r2τ u(τ)dτ + c2er2t . Note that the solution y(t) contains two arbitrary constants. 29. We have (D2 + 2D + 1)y = (D + 1)(D + 1)y . Let u = (D + 1)y , and consider the ODE u + u = 2e−t . The general solution is u(t) = 2t e−t + c e−t . We therefore have the first order equation u + u = 2t e−t + c1e−t . The general solution of the latter differential equation is y(t) = e−t t t0 [2τ + c1 ] dτ + c2e−t = e−t (t2 + c1t + c2).
  • 18. 70 Chapter 3. Second-Order Linear Equations 30. We have (D2 + 2D)y = D(D + 2)y . Let u = (D + 2)y , and consider the equa- tion u = 3 + 4 sin 2t . Direct integration results in u(t) = 3t − 2 cos 2t + c . The problem is reduced to solving the ODE y + 2y = 3t − 2 cos 2t + c . The general solution of this first order differential equation is y(t) = e−2t t t0 e2τ [3τ − 2 cos 2τ + c ] dτ + c2e−2t = = 3 2 t − 1 2 (cos 2t + sin 2t) + c1 + c2e−2t . 3.6 1. The solution of the homogeneous equation is yc(t) = c1e2t + c2e3t . The functions y1(t) = e2t and y2(t) = e3t form a fundamental set of solutions. The Wronskian of these functions is W(y1, y2) = e5t . Using the method of variation of parameters, the particular solution is given by Y (t) = u1(t) y1(t) + u2(t) y2(t), in which u1(t) = − e3t (2et ) W(t) dt = 2e−t and u2(t) = e2t (2et ) W(t) dt = −e−2t . Hence the particular solution is Y (t) = 2et − et = et . 3. The functions y1(t) = et/2 and y2(t) = tet/2 form a fundamental set of solutions. The Wronskian of these functions is W(y1, y2) = et . First write the equation in standard form, so that g(t) = 4et/2 . Using the method of variation of parameters, the particular solution is given by Y (t) = u1(t) y1(t) + u2(t) y2(t), in which u1(t) = − tet/2 (4et/2 ) W(t) dt = −2t2 and u2(t) = et/2 (4et/2 ) W(t) dt = 4t. Hence the particular solution is Y (t) = −2t2 et/2 + 4t2 et/2 = 2t2 et/2 . 5. The solution of the homogeneous equation is yc(t) = c1 cos 3t + c2 sin 3t. The two functions y1(t) = cos 3t and y2(t) = sin 3t form a fundamental set of solu- tions, with W(y1, y2) = 3 . The particular solution is given by Y (t) = u1(t) y1(t) + u2(t) y2(t), in which u1(t) = − sin 3t(9 sec2 3t) W(t) dt = − csc 3t u2(t) = cos 3t(9 sec2 3t) W(t) dt = ln(sec 3t + tan 3t), since 0 < t < π/6. Hence Y (t) = −1 + (sin 3t) ln(sec 3t + tan 3t). The general so- lution is given by y(t) = c1 cos 3t + c2 sin 3t + (sin 3t) ln(sec 3t + tan 3t) − 1 .
  • 19. 3.6 71 6. The functions y1(t) = e−2t and y2(t) = te−2t form a fundamental set of so- lutions. The Wronskian of these functions is W(y1, y2) = e−4t . The particular solution is given by Y (t) = u1(t) y1(t) + u2(t) y2(t), in which u1(t) = − te−2t (t−2 e−2t ) W(t) dt = − ln t and u2(t) = e−2t (t−2 e−2t ) W(t) dt = −1/t. Hence the particular solution is Y (t) = −e−2t ln t − e−2t . Since the second term is a solution of the homogeneous equation, the general solution is given by y(t) = c1e−2t + c2te−2t − e−2t ln t. 7. The functions y1(t) = cos (t/2) and y2(t) = sin(t/2) form a fundamental set of solutions. The Wronskian of these functions is W(y1, y2) = 1/2 . First write the ODE in standard form, so that g(t) = sec(t/2)/2. The particular solution is given by Y (t) = u1(t) y1(t) + u2(t) y2(t), in which u1(t) = − cos (t/2) [sec(t/2)] 2W(t) dt = 2 ln(cos (t/2)) u2(t) = sin(t/2) [sec(t/2)] 2W(t) dt = t. The particular solution is Y (t) = 2 cos(t/2) ln(cos (t/2)) + t sin(t/2). The general solution is given by y(t) = c1 cos (t/2) + c2 sin(t/2) + 2 cos(t/2) ln(cos (t/2)) + t sin(t/2). 8. The solution of the homogeneous equation is yc(t) = c1et + c2tet . The functions y1(t) = et and y2(t) = tet form a fundamental set of solutions, with W(y1, y2) = e2t . The particular solution is given by Y (t) = u1(t) y1(t) + u2(t) y2(t), in which u1(t) = − tet (et ) W(t)(1 + t2) dt = − 1 2 ln(1 + t2 ) u2(t) = et (et ) W(t)(1 + t2) dt = arctan t. The particular solution is Y (t) = −(1/2)et ln(1 + t2 ) + tet arctan(t). Hence the general solution is given by y(t) = c1et + c2tet − 1 2 et ln(1 + t2 ) + tet arctan(t). 10. Note first that p(t) = 0 , q(t) = −2/t2 and g(t) = (3t2 − 1)/t2 . The functions y1(t) and y2(t) are solutions of the homogeneous equation, verified by substitution. The Wronskian of these two functions is W(y1,y2) = −3 . Using the method of variation of parameters, the particular solution is Y (t) = u1(t) y1(t) + u2(t) y2(t), in which u1(t) = − t−1 (3t2 − 1) t2 W(t) dt = t−2 /6 + ln t
  • 20. 72 Chapter 3. Second-Order Linear Equations u2(t) = t2 (3t2 − 1) t2 W(t) dt = −t3 /3 + t/3. Therefore Y (t) = 1/6 + t2 ln t − t2 /3 + 1/3 . 12. Observe that g(t) = t e2t . The functions y1(t) and y2(t) are a fundamental set of solutions. The Wronskian of these two functions is W(y1,y2) = t et . Using the method of variation of parameters, the particular solution is Y (t) = u1(t) y1(t) + u2(t) y2(t), in which u1(t) = − et (t e2t ) W(t) dt = −e2t /2 and u2(t) = (1 + t)(t e2t ) W(t) dt = t et . Therefore Y (t) = −(1 + t)e2t /2 + t e2t = −e2t /2 + t e2t /2 . 13. Note that g(x) = ln x . The functions y1(x) = x2 and y2(x) = x2 ln x are solu- tions of the homogeneous equation, as verified by substitution. The Wronskian of the solutions is W(y1,y2) = x3 . Using the method of variation of parameters, the particular solution is Y (x) = u1(x) y1(x) + u2(x) y2(x), in which u1(x) = − x2 ln x(ln x) W(x) dx = −(ln x)3 /3 u2(x) = x2 (ln x) W(x) dx = (ln x)2 /2. Therefore Y (x) = −x2 (ln x)3 /3 + x2 (ln x)3 /2 = x2 (ln x)3 /6 . 15. First write the equation in standard form. The forcing function becomes g(x)/x2 . The functions y1(x) = x−1/2 sin x and y2(x) = x−1/2 cos x are a fun- damental set of solutions. The Wronskian of the solutions is W(y1,y2) = −1/x . Using the method of variation of parameters, the particular solution is Y (x) = u1(x) y1(x) + u2(x) y2(x), in which u1(x) = x x0 cos τ (g(τ)) τ √ τ dτ and u2(x) = − x x0 sin τ (g(τ)) τ √ τ dτ. Therefore Y (x) = sin x √ x x x0 cos τ (g(τ)) τ √ τ dt − cos x √ x x x0 sin τ (g(τ)) τ √ τ dτ = = 1 √ x x x0 sin(x − τ) g(τ) τ √ τ dτ . 16. Eq.(28) is Y (t) = −y1(t) t t0 y2(s)g(s) W(y1, y2)(s) ds + y2(t) t t0 y1(s)g(s) W(y1, y2)(s) ds, where t0 is now considered the initial point. Bringing the terms y1(t) and y2(t) inside the integrals and using the fact that W(y1, y2)(s) = y1(s)y2(s) − y1(s)y2(s), the desired result holds. To show that Y (t) satisfies L[y] = g(t) we must take the derivative using Leibniz’s rule, which says that if y(t) = t t0 G(t, s) ds, then
  • 21. 3.6 73 Y (t) = G(t, t) + t t0 Gt(t, s) ds. Letting G(t, s) be the above integrand, we have that G(t, t) = 0 and ∂G ∂t = y1(s)y2(t) − y1(t)y2(s) W(y1, y2)(s) g(s). Likewise, Y = ∂G(t, t) ∂t + t t0 ∂2 G ∂t2 (t, s) ds = g(t) + t t0 y1(s)y2 (t) − y1 (t)y2(s) W(y1, y2)(s) g(s) ds. Since y1 and y2 are solutions of L[y] = 0, we have L[Y ] = g(t) since all the terms involving the integral will add to zero. Clearly y(t0) = 0 and y (t0) = 0. 17. Let y1(t) and y2(t) be a fundamental set of solutions, and W(t) = W(y1, y2) be the corresponding Wronskian. Any solution, u(t), of the homogeneous equation is a linear combination u(t) = α1y1(t) + α2y2(t). Invoking the initial conditions, we require that y0 = α1 y1(t0) + α2 y2(t0) y0 = α1 y1(t0) + α2 y2(t0) Note that this system of equations has a unique solution, since W(t0) = 0. Now consider the nonhomogeneous problem, L [v] = g(t), with homogeneous initial con- ditions. Using the method of variation of parameters, the particular solution is given by Y (t) = −y1(t) t t0 y2(s) g(s) W(s) ds + y2(t) t t0 y1(s) g(s) W(s) ds . The general solution of the IVP (iii) is v(t) = β1y1(t) + β2y2(t) + Y (t) = β1y1(t) + β2y2(t) + y1(t)u1(t) + y2(t)u2(t) in which u1 and u2 are defined above. Invoking the initial conditions, we require that 0 = β1y1(t0) + β2y2(t0) + Y (t0) 0 = β1y1(t0) + β2y2(t0) + Y (t0) Based on the definition of u1 and u2 , Y (t0) = 0 . Furthermore, since y1u1 + y2u2 = 0 , it follows that Y (t0) = 0 . Hence the only solution of the above system of equa- tions is the trivial solution. Therefore v(t) = Y (t) . Now consider the function y = u + v . Then L [y] = L [u + v] = L [u] + L [v] = g(t). That is, y(t) is a solution of the nonhomogeneous problem. Further, y(t0) = u(t0) + v(t0) = y0 , and simi- larly, y (t0) = y0 . By the uniqueness theorems, y(t) is the unique solution of the initial value problem. 18.(a) A fundamental set of solutions is y1(t) = cos t and y2(t) = sin t . The Wron- skian W(t) = y1y2 − y1y2 = 1 . By the result in Problem 17, Y (t) = t t0 cos(s) sin(t) − cos(t) sin(s) W(s) g(s)ds = t t0 [cos(s) sin(t) − cos(t) sin(s)] g(s)ds .
  • 22. 74 Chapter 3. Second-Order Linear Equations Finally, we have cos(s) sin(t) − cos(t) sin(s) = sin(t − s). (b) Using Problem 16 and part (a), the solution is y(t) = y0 cos t + y0 sin t + t 0 sin(t − s)g(s)ds . 19. A fundamental set of solutions is y1(t) = eat and y2(t) = ebt . The Wronskian W(t) = y1y2 − y1y2 = (b − a)e(a+b)t . By the result in Problem 17, Y (t) = t t0 eas ebt − eat ebs W(s) g(s)ds = 1 b − a t t0 eas ebt − eat ebs e(a+b)s g(s)ds . Hence the particular solution is Y (t) = 1 b − a t t0 eb(t−s) − ea(t−s) g(s)ds . 21. A fundamental set of solutions is y1(t) = eat and y2(t) = teat . The Wronskian W(t) = y1y2 − y1y2 = e2at . By the result in Problem 17, Y (t) = t t0 teas+at − s eat+as W(s) g(s)ds = t t0 (t − s)eas+at e2as g(s)ds . Hence the particular solution is Y (t) = t t0 (t − s)ea(t−s) g(s)ds . 22. The form of the kernel depends on the characteristic roots. If the roots are real and distinct, K(t − s) = eb(t−s) − ea(t−s) b − a . If the roots are real and identical, K(t − s) = (t − s)ea(t−s) . If the roots are complex conjugates, K(t − s) = eλ(t−s) sin µ(t − s) µ . 23. Let y(t) = v(t)y1(t), in which y1(t) is a solution of the homogeneous equation. Substitution into the given ODE results in v y1 + 2v y1 + vy1 + p(t) [v y1 + vy1] + q(t)vy1 = g(t) . By assumption, y1 + p(t)y1 + q(t)y1 = 0 , hence v(t) must be a solution of the ODE v y1 + [2y1 + p(t)y1] v = g(t) . Setting w = v , we also have w y1 + [2y1 + p(t)y1] w = g(t) .
  • 23. 3.7 75 25. First write the equation as y + 7t−1 y + 5t−2 y = t−1 . As shown in Problem 23, the function y(t) = t−1 v(t) is a solution of the given ODE as long as v is a solution of t−1 v + −2t−2 + 7t−2 v = t−1 , that is, v + 5t−1 v = 1 . This ODE is linear and first order in v . The integrating factor is µ = t5 . The solution is v = t/6 + c t−5 . Direct integration now results in v(t) = t2 /12 + c1t−4 + c2 . Hence y(t) = t/12 + c1t−5 + c2t−1 . 26. Write the equation as y − t−1 (1 + t)y + t−1 y = t e2t . As shown in Problem 23, the function y(t) = (1 + t)v(t) is a solution of the given ODE as long as v is a solution of (1 + t) v + 2 − t−1 (1 + t)2 v = t e2t , that is, v − 1 + t2 t(t + 1) v = t t + 1 e2t . This equation is first order linear in v , with integrating factor µ = t−1 (1 + t)2 e−t . The solution is v = (t2 e2t + c1tet )/(1 + t)2 . Integrating, we obtain v(t) = e2t /2 − e2t /(t + 1) + c1et /(t + 1) + c2 . Hence the solution of the original ODE is y(t) = (t − 1)e2t /2 + c1et + c2(t + 1) . 3.7 1. R cos δ = 3 and R sin δ = 4, so R = √ 25 = 5 and δ = arctan(4/3). We obtain that u = 5 cos(2t − arctan(4/3)). 2. R cos δ = −2 and R sin δ = −3, so R = √ 13 and δ = π + arctan(3/2). We obtain that u = √ 13 cos(πt − π − arctan(3/2)). 4. The spring constant is k = 3/(1/4) = 12 lb/ft . Mass m = 3/32 lb-s2 /ft . Since there is no damping, the equation of motion is 3u /32 + 12u = 0, that is, u + 128u = 0 . The initial conditions are u(0) = −1/12 ft , u (0) = 2 ft/s. The general solution is u(t) = A cos 8 √ 2 t + B sin 8 √ 2 t . Invoking the initial conditions, we have u(t) = − 1 12 cos 8 √ 2 t + 1 4 √ 2 sin 8 √ 2 t . R = 11/288 ft, δ = π − arctan(3/ √ 2 ) rad, ω0 = 8 √ 2 rad/s, T = π/(4 √ 2 ) s. 6. The spring constant is k = 3/(.1) = 30 N/m . The damping coefficient is given as γ = 3/5 N-s/m . Hence the equation of motion is 2u + 3u /5 + 30u = 0, that is, u + 0.3u + 15u = 0 . The initial conditions are u(0) = 0.05 m and u (0) = 0.01 m/s. The general solution is u(t) = A cos µt + B sin µt , in which µ = 3.87008 rad/s . Invoking the initial conditions, we have u(t) = e−0.15t (0.05 cos µt + 0.00452 sin µt). Also, µ/ω0 = 3.87008/ √ 15 ≈ 0.99925 .
  • 24. 76 Chapter 3. Second-Order Linear Equations 8. The frequency of the undamped motion is ω0 = 1 . The quasi frequency of the damped motion is µ = 4 − γ2 /2. Setting µ = 2ω0 /3, we obtain γ = 2 √ 5 /3. 9. The spring constant is k = mg/L . The equation of motion for an undamped system is mu + mgu/L = 0. Hence the natural frequency of the system is ω0 = g/L . The period is T = 2π/ω0 . 10. The general solution of the system is u(t) = A cos γ(t − t0) + B sin γ(t − t0) . Invoking the initial conditions, we have u(t) = u0 cos γ(t − t0) + (u0/γ) sin γ(t − t0). Clearly, the functions v = u0 cos γ(t − t0) and w = (u0/γ) sin γ(t − t0) satisfy the given criteria. 11. Note that r sin( ω0t − θ) = r sin ω0t cos θ − r cos ω0t sin θ . Comparing the given expressions, we have A = −r sin θ and B = r cos θ . That is, r = R =√ A2 + B2 , and tan θ = −A/B = −1/ tan δ . The latter relation is also tan θ + cot δ = 1 . 12. The system is critically damped, when R = 2 L/C . Here R = 1000 ohms. 15.(a) Let u = Re−γt/2m cos(µt − δ). Then attains a maximum when µtk − δ = 2kπ. Hence Td = tk+1 − tk = 2π/µ . (b) u(tk)/u(tk+1) = e−γtk/2m /e−γtk+1/2m = e(γtk+1−γtk)/2m . Hence u(tk)/u(tk+1) = eγ(2π/µ)/2m = eγ Td/2m . (c) ∆ = ln [u(tk)/u(tk+1)] = γ(2π/µ)/2m = πγ/µm . 16. The spring constant is k = 16/(1/4) = 64 lb/ft . Mass m = 1/2 lb-s2 /ft . The damping coefficient is γ = 2 lb-s/ft . The quasi frequency is µ = 2 √ 31 rad/s . Hence ∆ = 2π/ √ 31 ≈ 1.1285 . 18.(a) The characteristic equation is mr2 + γr + k = 0 . Since γ2 < 4km , the roots are r1,2 = (−γ ± i 4mk − γ2)/2m. The general solution is u(t) = e−γt/2m A cos 4mk − γ2 2m t + B sin 4mk − γ2 2m t . Invoking the initial conditions, A = u0 and B = (2mv0 − γu0)/ 4mk − γ2 . (b) We can write u(t) = R e−γt/2m cos(µt − δ) , in which R = u2 0 + (2mv0 − γu0)2 4mk − γ2 and δ = arctan (2mv0 − γu0) u0 4mk − γ2 . (c) R = u2 0 + (2mv0 − γu0)2 4mk − γ2 = 2 m(ku2 0 + γu0v0 + mv2 0) 4mk − γ2 = a + bγ 4mk − γ2 .
  • 25. 3.7 77 It is evident that R increases (monotonically) without bound as γ → (2 √ mk)− . 20.(a) The general solution is u(t) = A cos √ 2 t + B sin √ 2 t . Invoking the initial conditions, we have u(t) = √ 2 sin √ 2 t . (b) (c) The condition u (0) = 2 implies that u(t) initially increases. Hence the phase point travels clockwise. 23. Based on Newton’s second law, with the positive direction to the right, F = mu , where F = −ku − γu . Hence the equation of motion is mu + γu + ku = 0 . The only difference in this problem is that the equilibrium position is located at the unstretched configuration of the spring. 24.(a) The restoring force exerted by the spring is Fs = −(ku + u3 ). The oppos- ing viscous force is Fd = −γu . Based on Newton’s second law, with the pos- itive direction to the right, Fs + Fd = mu . Hence the equation of motion is mu + γu + ku + u3 = 0 . (b) With the specified parameter values, the equation of motion is u + u = 0 . The general solution of this ODE is u(t) = A cos t + B sin t . Invoking the initial conditions, the specific solution is u(t) = sin t . Clearly, the amplitude is R = 1, and the period of the motion is T = 2π .
  • 26. 78 Chapter 3. Second-Order Linear Equations (c) Given = 0.1 , the equation of motion is u + u + 0.1 u3 = 0 . A solution of the IVP can be generated numerically. We estimate A = 0.98 and T = 6.07. (d) For = 0.2, A = 0.96 and T = 5.90. For = 0.3, A = 0.94 and T = 5.74. (e) The amplitude and period both seem to decrease. (f) For = −0.1, A = 1.03 and T = 6.55. For = −0.2, A = 1.06 and T = 6.90. For = −0.3, A = 1.11 and T = 7.41. The amplitude and period both seem to increase. (a) ε = −0.1 (b) ε = −0.2 (c) ε = −0.3
  • 27. 3.8 79 3.8 1. We have sin(α ± β) = sin α cos β ± cos α sin β . Subtracting the two identities, we obtain sin(α + β) − sin(α − β) = 2 cos α sin β . Setting α + β = 7t and α − β = 6t, we get that α = 6.5t and β = 0.5t . This implies that sin 7t − sin 6t = 2 sin (t/2) cos (13t/2). 2. Consider the trigonometric identities cos(α ± β) = cos α cos β sin α sin β. Adding the two identities, we get cos(α − β) + cos(α + β) = 2 cos α cos β . Com- paring the expressions, set α + β = 2πt and α − β = πt. This means α = 3πt/2 and β = πt/2 . Upon substitution, we have cos(πt) + cos(2πt) = 2 cos(3πt/2) cos(πt/2) . 3. Adding the two identities sin(α ± β) = sin α cos β ± cos α sin β , it follows that sin(α − β) + sin(α + β) = 2 sin α cos β . Setting α + β = 4t and α − β = 3t, we have α = 7t/2 and β = t/2 . Hence sin 3t + sin 4t = 2 sin(7t/2) cos(t/2) . 4. Using MKS units, the spring constant is k = 5(9.8)/0.1 = 490 N/m , and the damping coefficient is γ = 2/0.04 = 50 N-s/m . The equation of motion is 5u + 50u + 490u = 10 sin(t/2) . The initial conditions are u(0) = 0 m and u (0) = 0.03 m/s . 5.(a) The homogeneous solution is uc(t) = Ae−5t cos √ 73 t + Be−5t sin √ 73 t . Based on the method of undetermined coefficients, the particular solution is U(t) = 1 153281 [−160 cos(t/2) + 3128 sin(t/2)] . Hence the general solution of the ODE is u(t) = uc(t) + U(t). Invoking the initial conditions, we find that A = 160/153281 and B = 383443 √ 73 /1118951300 . Hence the response is u(t) = 1 153281 160 e−5t cos √ 73 t + 383443 √ 73 7300 e−5t sin √ 73 t + U(t). (b) uc(t) is the transient part and U(t) is the steady state part of the response.
  • 28. 80 Chapter 3. Second-Order Linear Equations (c) (d) The amplitude of the forced response is given by R = 2/∆, in which ∆ = 25(98 − ω2)2 + 2500 ω2 . The maximum amplitude is attained when ∆ is a minimum. Hence the amplitude is maximum at ω = 4 √ 3 rad/s . 8. The equation of motion is 2u + u + 3u = 3 cos 3t − 2 sin 3t. Since the system is damped, the steady state response is equal to the particular solution. Using the method of undetermined coefficients, we obtain uss(t) = (sin 3t − cos 3t)/6. Further, we find that R = √ 2 /6 and δ = arctan(−1) = 3π/4 . Hence we can write uss(t) = ( √ 2 /6) cos(3t − 3π/4). 9.(a) Plug in u(t) = R cos(ωt − δ) into the equation mu + γu + ku = F0 cos ωt, then use trigonometric identities and compare the coefficients of cos ωt and sin ωt. The result follows. (b) First note that since R = F0/∆, Rk/F0 = k/∆ and that since Γ = γ2 /(mk), (γ2 ω2 )/m2 = Γω2 0ω2 . Then using Eq.12, Rk F0 = k ∆ = mω2 0 m2(ω2 0 − ω2)2 + γ2ω2 = mω2 0 m2(ω2 0 − ω2)2 + γ2ω2 = ω2 0 (ω2 0 − ω2)2 + γ2ω2 m2 = ω2 0 (ω2 0 − ω2)2 + Γω2 0ω2 = 1 ω2 0 −ω2 ω2 0 2 + Γ ω2 0 ω2 ω4 0 = 1 1 − ω2 ω2 0 2 + Γω2 ω2 0 (c) The amplitude of the steady-state response is given by R = F0 m2(ω2 0 − ω2)2 + γ2 ω2 . Since F0 is constant, the amplitude is maximum when the denominator of R is minimum . Let z = ω2 , and consider the function f(z) = m2 (ω2 0 − z)2 + γ2 z . Note
  • 29. 3.8 81 that f(z) is a quadratic, with minimum at z = ω2 0 − γ2 /2m2 . Hence the amplitude R attains a maximum at ω2 max = ω2 0 − γ2 /2m2 . Furthermore, since ω2 0 = k/m , ω2 max = ω2 0 1 − γ2 2km . (d) Substituting ω2 = ω2 max into the expression for the amplitude R gives the maximum value for R: Rmax = F0 γ4/4m2 + γ2 (ω2 0 − γ2/2m2) = F0 ω2 0γ2 − γ4/4m2 = F0 γω0 1 − γ2/4mk . To understand the approximation, note that Rmax = F0 γω0 1 − γ2 4mk −1/2 Recall that binomial theorem states that (1 + a)p ≈ 1 + pa when a is small. Ap- plying this result with a = −γ2 /(4mk) and p = −1/2 gives that Rmax = F0 γω0 1 − γ2 4mk −1/2 ≈ F0 γω0 1 + − 1 2 − γ2 4mk = F0 γω0 1 + γ2 8mk 13.(a) The homogeneous solution is uc(t) = A cos t + B sin t . Based on the method of undetermined coefficients, the particular solution is U(t) = 3 1 − ω2 cos ωt . Hence the general solution of the ODE is u(t) = uc(t) + U(t). Invoking the initial conditions, we find that A = 3/(ω2 − 1) and B = 0 . Hence the response is u(t) = 3 1 − ω2 [ cos ωt − cos t ] . (b) (a) ω = 0.7 (b) ω = 0.8 (c) ω = 0.9 Note that u(t) = 6 1 − ω2 sin (1 − ω)t 2 sin (ω + 1)t 2 .
  • 30. 82 Chapter 3. Second-Order Linear Equations 14.(a) The homogeneous solution is uc(t) = A cos t + B sin t . Based on the method of undetermined coefficients, the particular solution is U(t) = 3 1 − ω2 cos ωt . Hence the general solution is u(t) = uc(t) + U(t). Invoking the initial conditions, we find that A = (ω2 + 2)/(ω2 − 1) and B = 1 . Hence the response is u(t) = 1 1 − ω2 3 cos ωt − (ω2 + 2) cos t + sin t . (b) (a) ω = 0.7 (b) ω = 0.8 (c) ω = 0.9 Note that u(t) = 6 1 − ω2 sin (1 − ω)t 2 sin (ω + 1)t 2 + cos t + sin t . 15. (a) ω = 0.7 (b) ω = 0.8 (c) ω = 0.9
  • 31. 3.8 83 18.(a) (b) Phase plot - u vs u :
  • 32. 84 Chapter 3. Second-Order Linear Equations Elementary Differential Equations 11th Edition Boyce Solutions Manual Full Download: https://alibabadownload.com/product/elementary-differential-equations-11th-edition-boyce-solutions-manual/ This sample only, Download all chapters at: AlibabaDownload.com